[ /tv/ /rf/ /vg/ /a/ /b/ /u/ /bo/ /fur/ /to/ /dt/ /cp/ /oe/ /bg/ /ve/ /r/ /mad/ /d/ /mu/ /cr/ /di/ /sw/ /hr/ /wh/ /lor/ /s/ /hau/ /slow/ /gf/ /vn/ /w/ /ma/ /azu/ /wn/ ] [ Main | Settings | Bookmarks | Music Player ]

No.137425 Reply
File: ОП-пик-Физика-тред-6.jpg
Jpg, 145.80 KB, 934×768 - Click the image to expand
edit Find source with google Find source with iqdb
ОП-пик-Физика-тред-6.jpg
Здесь, возможно, начнётся неспешное обсуждение топологических изоляторов.

Предыдущие обсуждения:
>>113576
>>88754
>>54970
>>16169
>>41
>> No.137430 Reply
Повторю вопрос: >>137423
>> No.137434 Reply
>>137425
> топологические изоляторы
Кто такие, чем знамениты?
>> No.137453 Reply
А вот вопрос к физикам, которые разбираются в предмете. Если мы возьмём пространство с метрикой, в которой более одной временной координаты, например (+,+,-,-), то к каким неприятностям, с формальной точки зрения, это приведёт? Кроме того, что хурр замкнутые времениподобные кривые.
>> No.137481 Reply
>>137453
Ни к каким. Смотри физику в пространствах (Анти)де Ситтера.
>> No.137519 Reply
>>137481
Анти де-Ситтер — аналог пространства Лобачевского (т.е. имеющий постоянную отрицательную кривизну), в котором одна из координат временная, с сигнатурой (-,+,+,+,...). Это не совсем то, о чём я говорю.
>> No.137546 Reply
File: re.png
Png, 33.57 KB, 500×850 - Click the image to expand
edit Find source with google Find source with iqdb
re.png
Доброчан, помоги! Никак не могу решить: "Определите оптимальную длину нити АВ".
>> No.137552 Reply
>>137546
Оптимальную для чего?
>> No.137553 Reply
>>137552
Тут идеальный шарик за идеальную ниточку подвешен к идеальной стенке идеальными креплениями. Радиус шарика - 1 м. Можно взять ниточку длиной 1000 м, а можно 10 м. Вопрос, какую брать?
>> No.137557 Reply
>>137553
Ну если наименьшей возможной длины, то кратчайшее расстояние от точки до плоскости - перпендикуляр.
>> No.137559 Reply
>>137557
Вектор *G* подсказывает, что ты меня обманываешь.
>> No.137560 Reply
Анон. Хочу сделать на даче ветряную мельницу, чтобы вырабатывала электричества.
Это вообще возможно?
>> No.137562 Reply
File: 505.jpg
Jpg, 102.29 KB, 605×922 - Click the image to expand
edit Find source with google Find source with iqdb
505.jpg
>>137425
Объясни тупому. Про электротехнику. В сети трёхфазного тока с изолированной нейтралью падает одна фаза на землю(например фаза B, в сети 10 кВ, замыкание пуская будет "идеальное", металлическое). Показания фазных киловольтметров будут такие: A=17,3 кВ; B=0 кВ; С=17,3 кВ. Верно же? На одной фазе напряжение падает до нуля, а на двух других возрастает до линейного. А теперь вопрос, какие будут показания линейного напряжения? A-B; B-C; C-A.
>> No.137589 Reply
File: Screenshot_2015-03-01-23-55-55.png
Png, 87.31 KB, 1200×1920 - Click the image to expand
edit Find source with google Find source with iqdb
Screenshot_2015-03-01-23-55-55.png
ну что же вы, физики? задачка первого курса машиностроительного
>>137546 нужно выразить зависимость реакции нити от её длины через угол, составить зависимость кривизны этой кривой от длины нити и найти её максимум
>> No.137592 Reply
>>137589
Я так и не понял, что именно требуется найти.
>> No.137593 Reply
>>137592
Я тоже. Доставьте определение оптимальности уже.
>> No.137594 Reply
>>137560
Молодец. Пили либо с вертикальной осью — сразу на земле, либо с парусными лопастями — относительно дёшево, пашет от слабого ветра.
Годится для перекачки ветра в тепло на ТЭНы, так как подобная схема не требует никаких преобразователей, накопителей и стабилизаторов. Ну можно теплоаккумулятор в виде бака с водой, а то и с парафином.
В качестве генератора в самопальных юзают моторы и автогенераторы.
>> No.137599 Reply
>>137592 требуется решить задачу о выборе длины нити для подвешивания шарика. решение должно быть оптимальным, критерии оптимальности извлечь из условия. для упрощения все объекты заданы идеальными.
наверное, это действительно слишком сложно. извините
>> No.137600 Reply
>>137599
> критерии оптимальности извлечь из условия
Да ты же нас тралируешь.
>> No.137602 Reply
>>137600 что, из чертежа уже никакая информация не извлекается?
>> No.137604 Reply
>>137602
Шар единичного радиуса с весом в 100 Н прикреплён к вертикальной стене нитью AB.
На этом информация, которую можно извлечь из чертежа, заканчивается.
>> No.137605 Reply
>>137604 очень плохо. геометрической информации достаточно, чтобы установить функции реакций опор и вычислить значения для любой длины нити. из этой же информации очевидно, что критериями оптимальности могут выступать лишь габариты конструкции и силы, действующие на опоры. далее проводится анализ кривых реакций, из которого следует, что после определённой точки, дальнейшее удлинение нити приводит лишь к увеличению габаритов, не давая никаких выгод - там ассимптота к G. в обратную сторону, укорочение нити приводит к резкому возрастанию реакций, что повышает требования к механической прочности опор. при нулевой длине нити, очевидно, не существует силы, способной удержать шар

задачка первого курса
>> No.137607 Reply
>>137605
> критериями оптимальности могут выступать лишь габариты конструкции и силы, действующие на опоры
Только вот функция (силы, размер) → цена может быть совершенно произвольной и в задаче не указана.
>> No.137616 Reply
Автомобиль начинает двигаться равноускоренно и достигает 20 м\с. Найти скорость на середине расстояния. В ответе написано - около 14 м\с, у меня столько не выходит, и вообще я понятия не имею, как считать, имея только один параметр.
>> No.137618 Reply
>>137616
Всё, посчитал, сошлось. Надо было взять ускорение за икс, выразить через него расстояние и идти по формуле V=(sqrt)2as
>> No.137624 Reply
>>137607 а где в задаче хоть слово про цену?
>> No.137625 Reply
>>137562
А что с моим линейным напряжением, анон? Или тут механика у вас процветает только
>> No.137629 Reply
>>137625 в сетях с изолированной нейтралью нет земли, их потому так и назвали. а тут школа процветает
>> No.137639 Reply
>>137624
Так что, собственно, оптимизируется?
>> No.137651 Reply
>>137629
> в сетях с изолированной нейтралью нет земли
Она только там и есть, "земля" в сети это "замыкание на землю", это не короткое замыкание, ты может это имел ввиду? При таком замыкании, защиты работают только на сигнал, фаза валяется, а определить на какой фазе обрыв, можно по киловольтметрам. Поэтому вопроса это моего никак не отменяет.
>> No.137658 Reply
>>137639 а чего оптимизирует топологический трассировщик?
>>137651 на питающей ничего не изменится, кроме появления у диспетчера сигнала утечки на землю. он не является сигналом о ЧП и носит информационный характер. у потребителя всё зависит от схемы соединения нагрузки. так какая схема у потребителя, как на твоей картинке? тогда пропадет только одна фаза, остальные не меняются
>> No.137671 Reply
>>137658
Ты тоже самое сказал, что и я. Это я всё знаю. Мне нужны показания киловольтметров, смотри вопрос >>137562
А схема для наглядности
>> No.137672 Reply
>>137658
А причём тут топологический трассировщик? В той задаче явно говорится: «найти оптимальную длину нити». Это подразумевает задачу оптимизации, которая сводится к нахождению экстремума некоторой функции. Однако, условие не предоставляет никакой информации о характере это функции.
К примеру, можно предположить, что цена крепления зависит от прочности экспоненциально, а первые 300 м нити бесплатны — это нисколько не противоречит условию задачи.
>> No.137676 Reply
Господа, можете просветить в напряженность электростатических полей, потенциал и напряженность в проводниках. Заранее премного благодарен.
>> No.137677 Reply
>>137676
Возьмём пространство, выберем в нём систему отсчёта. Предположим, что в пространстве есть несколько неподвижных электрических зарядов. Внесём в это пространство какое-нибудь заряженное тело, или, иначе говоря, внесём пробный заряд. Тогда это тело начнёт двигаться, и даже с ускорением. Точнее говоря, на тело будет действовать некая сила.

В таком вот случае, - когда в пространстве есть один или несколько покоящихся электрических зарядов, неизменных со временем, - мы будем говорить, что в пространстве есть электростатическое поле. Силу, действующую на внесённый заряд, мы будем называть силой, действующей со стороны электростатического поля. Обозначим эту силу буквой F. В разных точках пространства на пробное тело действуют, вообще говоря, разные силы.

Из опытов известно, что сила, с которой поле действует на пробный заряд, зависит не только от величины неподвижных зарядов электростатического поля, но и от величины пробного заряда. Чтобы избавиться от этой зависимости, введём понятие напряжённости: поделим силу, действующую со стороны поля на пробный заряд, на величину этого заряда. E = F/q.
Сила в каждой точке пространства своя, поэтому напряжённость поля также в каждой точке пространства своя.

Поскольку на пробный заряд со стороны электростатического поля действует сила, мы должны считать, что при движении заряда совершается какая-то работа. Если заряд перемещается из точки 1 в точку 2, то работу, которую поле совершает по перемещению заряда, мы будем обозначать A(1,2). Оказывается, что эта работа не зависит от пути, по которому двигался пробный заряд. Какими бы извилистыми не были два разных пути, работа A(1,2) для них одинаковая.

Поскольку сила, действующая на пробный заряд со стороны поля зависит, как я уже говорил, от величины этого заряда, совершаемая полем работа также зависит от величины пробного заряда. Чтобы избавиться от этой зависимости, введём понятие "разность потенциалов": разделим работу на величину заряда. Разность электростатических потенциалов между точками 1 и 2 есть работа, совершенная полем по перемещению пробного заряда из точки 1 в точку 2, делённая на величину этого пробного заряда.
ф(1,2) = A(1, 2)/q.
По-другому разность потенциалов между точками 1 и 2 называется напряжением между точками 1 и 2. Разность потенциалов и напряжение - это синонимы.

Потенциал точки определяется независимо от разности потенциалов. Выясняется, что потенциал первой точки минус потенциал второй точки (в таком порядке) действительно есть напряжение, то есть слово "разность" в названии напряжения не поэтическая метафора, а настоящее вычитание двух неких величин. О том, как именно определяются потенциал, проводники и всё остальное, сможешь прочитать в книжке http://mathus.ru/phys/book.pdf, например.
>> No.137683 Reply
>>137677
Еще раз спасибо, няша, спас меня от вылета ^^
>> No.137716 Reply
>>137671
Короче я сам додумался, всё просто. Векторная сумма фазных напряжений, даёт линейное. Когда одна фаза (например А) будет ноль, а 2 другие в 1,74 раз больше (т.е. возрастёт до линейного), то получается А-B=0, C-A=0 и B-C бдует равно сумме двух векторов(используя теорему косинусов), угол между которыми в 120 градусов (ведь фазы у нас различаются на 120 градусов), т.е. например в сети 10 кВ, напряжение там будет ~30 кВ.
>> No.137822 Reply
Не могу решать задачи. в теории всё просто, а задачи никак не идут
>> No.137853 Reply
>>137716 и тебя ни капли не смутило, что в статье речь шла о разности потенциалов?
>> No.137869 Reply
>>137853
В какой статье?
>> No.137871 Reply
>>137869 с сайтов типа электромонтер.инфо, откуда взято это повышение до линейного при обрыве: там говорится про обрыв на ВЛ
>> No.137877 Reply
>>137871
Да причём тут именно вл. Это замыкание может и в зру у тебя бахнуть в ячейке, суть не меняется. Я в учебнике это читал. Я вообще не понимаю что ты хочешь сказать? Ну разница потенциалов и что?
>> No.137878 Reply
>>137877 разность потенциалов есть между любыми двумя точками в пространстве. и ещё:
> в сети 10 кВ
а 10 кВ - это между чем и чем напряжение?
>> No.137883 Reply
>>137878
Да я не понял чего ты вообще это написал. Я знаю что это такое, не надо мне объяснять элементарные вещи, лол.
> а 10 кВ - это между чем и чем напряжение?
Это фазное напряжение, между фазой и нулём. Уже разобрались же во всём, чего тебе не понятно то, сформулируй нормально вопрос.
>> No.137892 Reply
>>137883 удачи на экзаменах, лол
>> No.137898 Reply
>>137892
Так что не так, ты тролишь что ли?
>> No.137998 Reply
Народ, а есть какие-нибудь интерактивные обучалки по физике?
>> No.137999 Reply
>>137998
Физикус же.
>> No.138237 Reply
Не знаю, где еще спросить: с помощью каких инструментов проводится разборка и починка генератора постоянного тока?
>> No.138240 Reply
все самые элементарные вопросы задаются перед дипломом
извините
>> No.138303 Reply
Так что делать с >>137423, физики?
>> No.138627 Reply
File: 14269769801460.jpg
Jpg, 46.65 KB, 604×453 - Click the image to expand
edit Find source with google Find source with iqdb
14269769801460.jpg
Berkeley Physics Course VS The Feynman Lectures on Physics VS Сивухин

Дано:
Математик, самостоятельно учу химию по Глинке, физику по Яковлеву, хочу хорошо понимать общефиз и потом плавно въехать в теорфиз (чтоб не стыдно было в магистратуру пойти учиться физике или в аспирантуре по матфизу угорать).
Найти:
Оптимальный курс общей физики уровня бакалавриата.
>> No.138631 Reply
>>138627
Ответ у тебя на пикрелейтед же. Серьёзно, ничего лучше не придумали.
>> No.138632 Reply
File: 14264653881422.jpg
Jpg, 265.32 KB, 531×797 - Click the image to expand
edit Find source with google Find source with iqdb
14264653881422.jpg
>>138631
Но это же теоретическая, а меня интересует общая физика.
>> No.138638 Reply
>>138632
Нет физики кроме теоретической, и Ландау пророк её.
>> No.138661 Reply
>>138627
А тебе зачем?
Можешь попробовать задачники Овчинкина (вроде бы, там есть теоретические врезки). Прорешаешь их целиком - сможешь решать задачки по общефизу уровня бакалавриата. После этого достаешь Коряавова - на случай, если лень думать.
Если хочется ощущения, что ты чего-то понял - читай Фейнмана. Если хочешь быть способным решать задачки (что и является, по-моему, пониманием), то практикуйся на задачках обязательно. Берклевский курс, вроде бы, неплох - нам его препод по термодинамике рекомендовал. Сивухина за годы обучения открывал пару раз, да и то на экзамене, не впечатлился.

Теорфиз это другое, и квантмех в ландафшице, вроде бы, неплохой. Без квантмеха в электронике будет сложновато, все в кристаллах идет оттуда, лазеры тоже.
Перед теорфизом освой теормех, Лагранжевы и Гамильтоновы формализмы, вот это все. От себя рекомендую Айзермана как теорию + методичку Трухан по задачам. Теормех это больше математика, чем физика, но расширяет мышление о том, что такое координаты и силы, а главное - дает методы, которые потом используются в теорфизе.
>> No.138670 Reply
>>138661
> зачем
Любопытство. Да, я из любопытства хочу пройти вузовский курс физики.

Спасибо за подробный ответ!
>> No.138671 Reply
File: 1381922203946.jpg
Jpg, 96.61 KB, 700×933
edit Find source with google Find source with iqdb
1381922203946.jpg
File: 14264653881391.jpg
Jpg, 127.85 KB, 600×800
edit Find source with google Find source with iqdb
14264653881391.jpg

>>138661
Держи няшку!
>> No.138728 Reply
>>138627
> учу химию по Глинке
зачем тебе химия школоуровня, если ты интересуешься физикой?
>> No.138766 Reply
File: 2015-03-29_16-00-...
Png, 12.01 KB, 514×149
edit Find source with google Find source with iqdb
2015-03-29_16-00-44.png
File: tumblr_nlineqgWBb...
Jpg, 125.72 KB, 500×573
edit Find source with google Find source with iqdb
tumblr_nlineqgWBb1rl1jado1_500.jpg

Котоны, вот такая задача. Свой учебник изрыл, не смог найти достойных подсказок. Надо найти плотность вероятности времени жизни, а потом усреднить. Может у кого есть учебник годный по молекулярке?
>> No.138914 Reply
Какой учебник посоветуете для самообучения с нуля?
>> No.138982 Reply
>>138914
Ландсберг жи есть.
>> No.138998 Reply
>>138671
Я бы её потрахал в ротик, а кончил бы в горло, чтобы она всё проглотила. Представляю её слёзы, но она всё равно продолжала его держать в себе, понимая, что в противном случае она просто умрёт. Безвыходность толкает людей на крайности. А меня толкнула бы необузданная сексуальность.
>> No.139020 Reply
>>138982
Спасибо.
>> No.139042 Reply
>>138728
Проебал полностью школьный курс, учу с нуля. Да и нужно мне её глубоко понимать и хорошо знать.
>> No.139305 Reply
File: 1f8d34d19ed3c9578c8f45044deed19f05b0f9ae.jpg
Jpg, 566.34 KB, 600×1217 - Click the image to expand
edit Find source with google Find source with iqdb
1f8d34d19ed3c9578c8f45044deed19f05b0f9ae.jpg
Может кто посоветовать чего по задачам на вакуумные ртутные насосы? Есть какая статейка или из учебника параграф? Нагуглил только общие принципы, конструкции и такая всякая хуйня. А мне бы именно что-нибудь связанное с задачами и разряженными газами.
>> No.139321 Reply
Ванабиполиматх итт. Такие простые, казалось бы, вопросы:
А сколько тут, собственно, людей со степенями выше бакалавриата по научно-математическим специальностям?
Есть ли, на самом деле, смысл начинать изучение наук с элементари левела, то есть "ну короче, если не углубляться в математику, basically оно вот так" с полным игнором истории развития теорий, что, как мне кажется, сильно сбивает толку даже с самыми практичными целями изучения на уме? Полупопсай эдакий. Школьный уровень, как принято называть. Или все таки въебать матёшки и сразу погрузиться в суть учебниками с университетов?
Сейчас многие кукарекают о том, что, дескать, всего уже знать нельзя - даже базово, слишком много знаний. Но таки возможно ли имея социальную жизнь освоить, скажем, физику, химию, биологию и нейронауки хотя бы на уровне бакалавра? Скажем, за пять лет. Потому что сдается мне, что всё это от человеческой лени и прокрастинации исходит, которые потом рационализируются в "сейчас уже всё в один томик не вместишь, это нереально" и классическое планктоновское "я просто слишком занятый/уставший человек". А вот если взять и перестать заниматься хуйней - оставив, конечно, пару увлечений по несколько часов в день в среднем, забыть про концепцию выходных, каникул и отпусков, и, блядь, учить. Разумеется, это при отсутствии работы или вузика. С вузом, забирающим до 2/3 дня, понятно, выйдет дольше. А непосредственно учится в нем пусть быдло, мне чужим ртом пережеванная пища не нужна принципиально, да и подход совершенно плебейский. Только ради оборудования для экспериментов и бумажки. И я даже не уверен на счет последней - продают же в переходах, как все говорят.
>> No.139324 Reply
File: э.jpg
Jpg, 90.33 KB, 601×720 - Click the image to expand
edit Find source with google Find source with iqdb
э.jpg
>>139321
За пять лет ты от силы выучишь одну физику и все. Ну если ты намерен ее выучить норм, а не на уровне "я знаю, что такое энтропия, а вы Львы Толстые". Не знаю как в биологии, а для физики надо много чего прочитать, срсли дохуя (думаю ты и сам представляешь). Придется параллельно учить матан, не слишком глубоко, но время займет. Плюс еще задач типовых нарешать и желательно со всякими приборами познакомиться.
Лучше выбирать что-нибудь одно, и дрочить это. Алсо, надо иметь ввиду, что у тебя вполне может все не сразу получаться.
> всё это от человеческой лени
Лень тут причем, если не можешь выучить механику за 3-6 месяца.
>> No.139343 Reply
>>139324
> Лень тут причем, если не можешь выучить механику за 3-6 месяца.
А много людей пытаются?
>> No.139361 Reply
>>139321
> А сколько тут, собственно, людей со степенями выше бакалавриата по научно-математическим специальностям?
Ну я аспирант-математик, например.
> Есть ли, на самом деле, смысл
Смысл для чего? В твоём вопросе не достаёт одной важной части - а зачем тебе всё это нужно? Какая цель?
Если цель - стать у мамки молодцом-полиматхом, то это очень слабая мотивация, на самом деле.
> Но таки возможно ли имея социальную жизнь освоить, скажем, физику, химию, биологию и нейронауки хотя бы на уровне бакалавра?Скажем, за пять лет.
На этот вопрос можешь ответить только ты сам. Берёшь программу гос. экзаменов по интересующей тебя специальности, сравниваешь объём вещей, уже тебе известных в этой программе, с объёмом неизвестных, экстраполируешь свой предыдущий опыт изучения этой дисциплины, понимаешь, сколько тебе нужно времени. Если ты пока не начал эту дисциплину изучать и не знаешь из программы ничего, то такие вопросы тебе задавать рано.
> "ну короче, если не углубляться в математику, basically оно вот так"
Ты ни в одном вузе не получишь степени бакалавра по физике, не углубляясь в математику. Да и по химии тоже. Я понимаю, что в вуз ты ходить не хочешь(почему-то), но единственный заданный тобой конкретный goalpost - получение бакалавра, а с ним дело обстоит именно так.
> имея социальную жизнь
> оставив, конечно, пару увлечений по несколько часов в день в среднем
Несколько шизофренично выглядит - ты сначала критикуешь "человеческую лень и прокрастинацию", а потом начинаешь оговариваться. Типа, ты, конечно, очень упорный, но на самом деле всё время посвящать наукам не готов, а хочешь ещё погулять с друзьяшками, а потом ещё у тебя будут увлечения на "несколько часов в день". Думаю, ты должен понимать, что твои планы весьма амбициозны, и тебе, скорее всего, придётся многим пожертвовать. В том числе, вероятно, и социальной жизнью, и другими увлечениями.
>> No.139396 Reply
>>139361
> а зачем тебе всё это нужно? Какая цель?
Чтобы знать, очевидно же. Ну и применения ирл не повредят, конечно. Зная нейронауку будет элементарно отделять маняпсихиатрию от психиатрии, не говоря уже о психологии. Химия может дать мне неплохой ворк проспект для заработка (планирую ебашить пхд в теор. физике, а на рамены жить не хочется).
> Если цель - стать у мамки молодцом-полиматхом, то это очень слабая мотивация, на самом деле.
Мотивация не имеет значения.
> На этот вопрос можешь ответить только ты сам.
> Если ты пока не начал эту дисциплину изучать и не знаешь из программы ничего, то такие вопросы тебе задавать рано.
То есть когда будет не рано, будет уже не нужно. Ты не Морфеус случайно? Я именно для того и спрашиваю, чтобы получить некий средний показатель и исходить из него, а как изучу достаточно, сравню скорости на пройденной дистанции и смогу сделать эстимацию своей итоговой скорости.
> Ты ни в одном вузе не получишь степени бакалавра по физике, не углубляясь в математику. Да и по химии тоже.
Вообще меня не понял. Я поднимаю вопрос бесполезности элементарных книжек, учитывая отсутствие в них математики и оторванность от хронологии научного прогресса.
> но единственный заданный тобой конкретный goalpost - получение бакалавра, а с ним дело обстоит именно так.
Бумажка мне, если что, на все дисциплины не нужна, только на ту, в которой буду науку ебашить. Верить в то, что бумажка отражает уровень знаний и уж тем более понимания - это как в бобро и мир во всем мире верить. Нет, с моими методами я буду знать гораздо больше типичного бакаврика.
> ты сначала критикуешь "человеческую лень и прокрастинацию", а потом начинаешь оговариваться.
Нонсенс. Лень и прокрастинация != потребность в ебле и других увлечениях.
> Типа, ты, конечно, очень упорный, но на самом деле всё время посвящать наукам не готов
Посвящать все время чему-то одному очень плохая идея по многим причинам. Всегда нужно иметь хотя бы два основных жизненных увлечений.
> а хочешь ещё погулять с друзьяшками, а потом ещё у тебя будут увлечения на "несколько часов в день". Думаю, ты должен понимать, что твои планы весьма амбициозны, и тебе, скорее всего, придётся многим пожертвовать.
Понимаю, конечно. Поэтому эти "несколько часов" весьма гибки в моем виденьи.
> В том числе, вероятно, и социальной жизнью, и другими увлечениями.
По идее - особенно если найти постоянную тлю - и преследовать только одно другое увлечение, времени должно хватить. Конечно, надо учесть и свои человеческие слабости, из-за которых порой неизбежно будет "да хули, норм же успеваю, можно и в йобу погонять пол часика".
>> No.139419 Reply
>>139396
Сори, полностью не читал, но крч вот как это делается (да, это делается): закрываешь борды, расклеиваешь на стенах плакаты в стиле "въебывай как сука отдыхай как мехмат" и ботаешь минимум по 8 часов каждый день. Не отвлекаясь, полностью концетрируясь на сабже. Лишние мысли лезут? Думаешь о тне, погоде, путене? Значит нужно больше мотивации. Разберись с диетой. Еда должна быть полезной (гугли форумы про ноотропы, там люди часто шарящие), не слишком много энергии должно уходить на усваивание. Займись спортом. Бег, плавание, в таком духе. Дает прилив сил и энергии. Делай всё по плану. Ставь цель на день/неделю/месяц и иди к ней. Для социальной жизни и прочей хуиты выдели несколько часов в один день недели. Это в основном опыт других людей, которые за пару семестров заканчивали бакалавриат.

Аудиокниги, ноотропы, здоровый сон, спорт, диета, полно отсутствие прокрастинации, сильная мотивация, планы и хардкорно въебывать 8-10 часов каждый день, полностью концентрируясь, и можно года за 2-3 получить 3-4 диплома.
>> No.139421 Reply
>>139419
Еще можно высверлить дырку в стене и нарисовать бабу вокруг. А еще можно пятилитровую баклашку воды налить и качаться ей. Ну и прочие советы.

мимо-скептик
>> No.139455 Reply
>>139419
> закрываешь борды
Да, все эти идиотские вопросы и просьбы советов исключительно от неуверенности.
> ботаешь минимум по 8 часов каждый день
У меня вот как раз эта цифра на уме и была. Допустим час на жратву и экскринацию, час на готовку/посуду, пол часа на уборку и 30-45 мин на слежку за собой, потому что это не каждый день делается, и остается 3 часа на что угодно. Это можно и тлю выебать дважды, и физкультурой заняться, и останется часа полтора на второе увлечение чередующееся с би. И при этом 8 часов сна.
> Значит нужно больше мотивации.
Я не верю в мотивацию как таковую. Уверен, что - также как и с уверенностью - это лишь отсутствие чего-то существующего. Как уверенность - это отсутствие неуверенности (производной тревоги), так и мотивация это отсутствие антимотивации, которая может существовать во множестве форм, но по сути сводится к руминации - всем этим ёбаным эмоциям, настроениям и состояниям.
У всех есть мотивация, фундаментально - максимизировать позитивные чувства и минимизировать негативные, откуда исходит глобальная цель побольше достичь в жизни, что бы это ни было. Гораздо эффективнее просто уничтожить свой механизм лени или в чем там проблема, чем дешевыми средствами прибавлять мотивации, что будет в лучшем случае слабенько тянуть тебя на буксире пару недель, пока не сдуется.
> гугли форумы про ноотропы, там люди часто шарящие
Орлы? Если ты про скептиков на доске, то сомневаюсь, что они так уж прям шарят в диетологии. Если про неАИБные форумы, то это один из самых тупых видов людей в интернете. Это как антивсдшников с их гомеопатией и религией в пример приводить.
> Займись спортом.
Тоже думаю.
> Для социальной жизни и прочей хуиты выдели несколько часов в один день недели.
Никак не годится, мне же надо быть дохуя лидером стаи, out there, the guy'ем. Хотя я понятия не имею как часто в тех же вузах собираются алкашить и что они еще там делают.
> Аудиокниги
Хуйня, нужно двигаться со своей скоростью, в своем ритме. Аудио просто неудобно для того чтобы взять и остановиться, чтобы самому дедуктировать всевозможные импликации, о которых расскажут позже, и в целом "поиграться" с этим новым куском информации, наебашить всяких интересных веток, пережевать и усвоить как следует нямку для ума.
> ноотропы
Ниверю. В любом случае, их только периодами можно пить, бред-н-баттером только кофеин является, но и там нужно быть очень аккуратным, знать, что делаешь, чтобы толер не рос, весь потенциал выжимать.
> и можно года за 2-3 получить 3-4 диплома.
В смысле настоящие бумажки, чтоле? То есть ты говоришь, что можно вот так просто придти на финальный экзамен/защиту бакалавриата и если зачтут, то всё, ты уже бакалавр? Или ты просто заочки имеешь в виду?
>> No.139516 Reply
Физик-куны, поясните, пожалуйста, за СРТ-инвариантность. Классический эксперимент Ву - что конкретно ожидали увидеть физики и что пошло не так? Сам эксперимент выглядит не очень сложным по сути, но я всё-таки умудрился запутаться в двухсоснахсреди этих чётностей. Сначала наблюдали распад кобальта-60 в магнитном поле, которое ориентировало ядра кобальта определённым образом. Ожидали найти равномерное распределение вероятности обнаружить электрон, но нашли, что в каком-то направлении эта вероятность выше, чем в других.
Иинвертировали магнитное поле, как бы отразив всю систему в зеркале и ожидая... Обнаружить что? Что распределение вероятностей по углам останется тем же, а оно изменилось, как в зеркале? Или что?
Что не так в данном случае с чётностью? Сам факт её изменения? Если ψ(-r, ...) = ψ(r, ...), то функция чётная и значит, отражение координат не должно было повлиять на вероятность обнаружения электронов в том же выделенном направлении? Но на деле всё оказалось иначе и ψ(-r, ...) = -ψ(r, ...), т.е. чётность изменилась, хотя для определения амплитуды вероятности ведь нужен квадрат волновой функции. Как изменение чётности описывает изменение выделенного направления электронов? СР-чётность Ландау должна была спасти сохранение симметрии только в случае радиоактивного бета-распада? ну, тогда наверно думали, что это универсальная затычка, а оказалось, что работает только для него И я не совсем понимаю, а как комбинированная чётность помогает здесь. Р-симметрия нарушена в любом случае. Без учёта, учитывается или нет зарядовое сопряжение же.
Извините за ламерство и возможно дубовую постановку вопросов, а также за ошибки. Но мне очень хотелось бы понять
>> No.139544 Reply
>>139419
> и ботаешь минимум по 8 часов каждый день
И сразу фейл. С такой длительностью, да ещё и без перерывов, мозг превращается в овощ и становится совершенно бесполезен.
Вот лекция на тему того, как правильно учиться: YouTube: Marty Lobdell - Study Less Study Smart
>> No.139563 Reply
>>139544
Лол, спс, я итак умею учиться Mind for numbers или как-то так неплохая книга по теме, хотя в сущности это мотивацию + общие идеи нейропсихологии, которую лучше по другим книжкам познавать. Мб биологическая особенность, но 5-6 часов без перерывов — норм. Потом сделать кофе, покурить и можно ещё часа 2 выжать без особых проблем, и только после этого концентрация смазывается. Тогда можно амфетаминов навернуть.
>>139455
Короче не будешь ты ничего никогда учить, только мечтать и рассуждать как было бы неплохо будешь.
>> No.139564 Reply
>>139563
А, ну разве что периодически сносит крышу и клинит мозг, иногда на несколько недель, но это тоже решаемые проблемы.
>> No.139573 Reply
File: tumblr_nch0t4yA1K...
Gif, 1026.94 KB, 500×355
edit Find source with google Find source with iqdb
tumblr_nch0t4yA1K1tdl796o1_500.gif
File: 2015-04-18_23-21-...
Png, 30.85 KB, 636×375
edit Find source with google Find source with iqdb
2015-04-18_23-21-52.png

Что за преобразования там юзаются? Как получается последнее уравнение? Спасай анон, я просто охуеваю с этих очевидностей.
Прикладываю учебник сам, страница 186.
http://rghost.ru/7XQXJKMwd
>> No.139597 Reply
>>139563
> Мб биологическая особенность, но 5-6 часов без перерывов — норм
И никакой усталости? Да ты монстр.
> Тогда можно амфетаминов навернуть.
Может, стоит пожалеть свой мозг?
>> No.139616 Reply
>>139563
> Короче не будешь ты ничего никогда учить, только мечтать и рассуждать как было бы неплохо будешь.
Мимо, еще летом начал и проблем с дисциплиной нет.
>> No.139624 Reply
>>139616
8 месяцев. И в чем ты уже бакалавра получил? Вангую максимум 1 курс по единственному направлению прошел.
>>139597
Усталость не повод не ебашить науки.
>> No.139626 Reply
>>139624
Всякое читал, в основном психулогию и подобный софткор. Сейчас вот когда с этим в общем-то закончил, пришла пора ебашить серьезное дерьмо.
>> No.139853 Reply
File: 2015-04-29_17-34-...
Png, 1.38 KB, 185×77
edit Find source with google Find source with iqdb
2015-04-29_17-34-19.png
File: 2015-04-29_17-34-...
Png, 5.89 KB, 758×75
edit Find source with google Find source with iqdb
2015-04-29_17-34-36.png

Чому выполняется пикрилейтед? Тот что маленький. Может кто поподробнее написать?
ruwiki://Внутренняя_энергия
>> No.139855 Reply
File: 14297056469320.jpg
Jpg, 58.06 KB, 419×604 - Click the image to expand
edit Find source with google Find source with iqdb
14297056469320.jpg
Физик-тян, помоги пожалуйста с парой вопросов по теории постоянного тока:
> 1. Что показывает разность потенциалов идеальный вольтметр, подключенный к клеммам источника тока в замкнутой и разомкнутой цепи.
> 2. Напишите формулы для последовательного/параллельного/встречного соединения источников тока.
> 3. Приведите схему зарядки аккумулятора и напишите формулу для напряжения зарядного устройства.
Заранее благодарен!

Капча: обычный поэткун
>> No.139856 Reply
>>139855
Источник тока идеальный?
Далее для идеального источника тока:
> источника тока
> разомкнутой цепи
на ноль поделил, няша. Не может быть разомкнутой цепи для идеального источника тока.
> в замкнутой
0. Ток есть, сопротивления нет...
> Напишите формулы
> параллельного
Сумма, но эквивалентный источник тока должен быть включён на цепь.
> последовательного
Низзя.
> встречного соединения источников тока.
Параллельное или последовательное? В любом случае, смотри выше.

электроэнергетик-кун
>> No.139972 Reply
Доброаноны, я тупая макака даже с гуглом: как формально определяется центр тяжести? Где я находил, либо что-то на пальцах, либо формулы, но в однородном поле тяжести, а хотелось бы, чтоб на куски тела влияли разные g.
>> No.139973 Reply
>>139972
Центром тяжести механической системы называется точка, относительно которой суммарный момент сил тяжести, действующих на систему, равен нулю.
>> No.139977 Reply
File: image.jpg
Jpg, 396.27 KB, 1272×1784
edit Find source with google Find source with iqdb
image.jpg
File: image.jpg
Jpg, 639.55 KB, 1271×1337
edit Find source with google Find source with iqdb
image.jpg

>>139853
Сивухин.
>> No.139989 Reply
File: 2015-05-05_10-28-17.png
Png, 15.62 KB, 384×514 - Click the image to expand
edit Find source with google Find source with iqdb
2015-05-05_10-28-17.png
>>139977
Пасибо! А разве нельзя как-то проще. Я думаю это можно сделать с помощью соотношений Максвелла. Пикрилейтед.
>> No.140016 Reply
>>139856
> разомкнутой цепи
Показывает ЭДС источника.
> в замкнутой
По закону ома U = (R{внешнее} + R{внутренее})*I
>> No.140031 Reply
Как именно бумеранг закручиваться это примерно понятно (как крученый мяч в бейсболе), а вот может ли бумеранг вернуться в руку владельца после удара?
Возьмем идеальные условия, открытую, безветренную местность, неподвижную цель. Отскок от абсолютно упругой мишени не считаться.
>> No.140115 Reply
Доброанон, помоги. Голова не соображает.

В магнитном поле индукцией 1.5Т движется проводник неизвестной длины и сопротивления, под воздействием внешней силы 5Н, со скоростью 10 м/с. На него действует тормозная электромагнитная сила и концам присоединён резистор сопротивлением 0.35 Ом. Под воздействием ЭДС в проводнике возникает ток 10 А.
Найти длину, сопротивление проводника, тормозную силу и ЭДС.
>> No.140118 Reply
Хотел тут спросить, вот по информатике была программка, где можно было создавать сети и т.п. роутеры ставить, соединять их... Может быть есть такая же программка только для электрических цепей? Там бы можно было бы делать свою электрическую цепь, диоды вешать, ключи замыкать и смотреть что будет?
>> No.140138 Reply
>>140118
http://cxem.net/software/soft_CAD.php
1 минута с гуглом. Неблагодари.
мимо
>> No.140159 Reply
Тело равномерно движется по окружности. Как изменится его угловая скорость, если радиус увеличить в 4 раза, а центростремительное ускорение оставить неизменным?
>> No.140190 Reply
>>140159
На этот вопрос уже есть ответ.


А как решить такое: "Пропеллер самолета радиусом R=1,5 м вращается с частотой n=2000 об/мин. Скорость самолета относительно земли U=162 км/ч. Определить скорость W точки на конце пропеллера. Ответ дать в м/с (округлить до целых)."
Значится, скорость пропеллера относительно самолёта 18840 м/мин, а скорость самолёта 2700 м/мин. Как высчитать относительно земли-то? В решении написано, что применять Пифагора - откуда там вылез Пифагор?
>> No.140195 Reply
File: Безымянный.png
Png, 64.78 KB, 807×476 - Click the image to expand
edit Find source with google Find source with iqdb
Безымянный.png
Нужна помощь с задачей.
Как будут распределены гауссовы поверхности? Так, как на втором рисунке красным пунктиром?
Как влияет центральный заряд q на напряженность в каждой из областей?
>> No.140200 Reply
File: vtk.jpg
Jpg, 4.50 KB, 233×139 - Click the image to expand
edit Find source with google Find source with iqdb
vtk.jpg
>>140190
> В решении написано, что применять Пифагора - откуда там вылез Пифагор?
Ну хоть складывать перпендикулярные стрелочки ты умеешь?
>> No.140345 Reply
>>140200
И где здесь пифагор?
>> No.140348 Reply
>>140345
Ты не умеешь складывать векторы?
>> No.140353 Reply
>>140190
Скорости идут вдоль разных прямых. Так что надо рисовать векторы и представлять, как они выглядят и как выглядит их сумма. После думать, как посчитать длину суммы векторов.
>> No.140381 Reply
File: рис1.jpg
Jpg, 57.46 KB, 451×486
edit Find source with google Find source with iqdb
рис1.jpg
File: рис2.jpg
Jpg, 37.60 KB, 344×400
edit Find source with google Find source with iqdb
рис2.jpg

Не могу понять, почему изменяется направление линий магнитного поля при внесении в пространство ферромагнетика. Например, читаю про трансформатор (рис.1): "... Так как обе обмотки надеты на общий железный сердечник, то почти все линии этого поля проходят через обмотки. Иначе можно сказать, что обе обмотки пронизываются одним и тем же магнитным потоком." Но куда девается то поле, которое я обозначил на рисунке красным? Да и с правой стороны поле ведь не всё "укладывается" в сердечник. Вернее, видимо, так и происходит, но я не понимаю почему.

То же самое наблюдается в случае магнитной экранировки (рис.2). "Линии индукции внешнего магнитного поля при переходе из среды (с низкой магнитной проницаемостью) в материал экрана (с высокой проницаемостью) заметно сгущаются, а в полости цилиндра густота линий индукции уменьшается, т.е. поле внутри цилиндра оказывается ослабленным." Тут мне опять непонятно, почему линии сгущаются, то есть меняют направление. Помогите разобраться, пожалуйста.
>> No.140687 Reply
>>137425
Товарищи физики, возник совершенно праздный вопрос: вращается ли воздух вместе с колесом автомобиля при движении?
>> No.140690 Reply
>>140687
И да и нет. Чем ближе к колесу, тем больше увлекается трением. Но это очень тонкий слой. Попробуй что-то статичное раскрутить и сунь в столб света на солнце, удостоверишься воочую на ходе пылинок что куда.
>> No.140692 Reply
>>140381
Внешнее поле заставляет упорядочиваться магнитные домены это ведь так называется? внутри ферромагнетика так, что поле самого ферромагнетика внутри него приобретает направленность, такую же, как у внешнего поля внутри ферромагнетика, а вне его линии, соответственно, имеют обратную направленность. Результирующее "искаженное" поле - это сумма двух полей. Оно сильнее, чем внешнее, внутри ферромагнетика, т.к. линии сонаправлены, и слабее снаружи, так как поле ферромагнетика забивает собой внешнее поле.
>> No.140695 Reply
>>140190
> Определить скорость W точки на конце пропеллера.
Относительно земли?
Но она же разная в разные моменты времени.
>> No.140696 Reply
>>140695
По модулю одинаковая.
>> No.140769 Reply
Уважаемый анон-физик, подскажи мне, пожалуйста, если не сложно.
Я репетиторствую понемногу, но хотелось бы взять дополнительный предмет - физику.
В школе и универе соображал на твёрдую 4.
Мне бы учебники хорошие, где доступным языком всё написано.
Знаешь такие? Особенно для подготовки к всяким ОГЭ и ЕГЭ
Или лучше так, с чего вообще начать изучать физику заново?
>> No.140788 Reply
Здравствуйте. Посоветуйте, пожалуйста, сборник олимпиадных задач по механике (10 класс). Вроде в школьной механике всё легко, но нужно убедиться всё же в своих силах.
>> No.141498 Reply
File: chibi_misaka_by_zraty-d30t4vo[1].png
Png, 86.05 KB, 885×903 - Click the image to expand
edit Find source with google Find source with iqdb
chibi_misaka_by_zraty-d30t4vo[1].png
Давайте порешаем задачки, что-ли. Вот моя:
Пусть атом гелия движется с некоторой скоростью в магнитном поле. При достаточно больших скорости атома и напряженности поля, атом должно разорвать силой Лоренца на электроны и ядро. Нужно найти требуемую напряженность и скорость (их соотношение), при которых это произойдет.
>> No.141930 Reply
File: u9jJzEz4aL4.jpg
Jpg, 71.71 KB, 1280×1024
edit Find source with google Find source with iqdb
u9jJzEz4aL4.jpg
File: Nci-vol-2268-300_...
Jpg, 1861.93 KB, 2700×1800
edit Find source with google Find source with iqdb
Nci-vol-2268-300_argon_ion_laser.jpg

Анон, поясни как работает лазер.
На уровне отдельных атомов я понимаю: электроны внешней силой поднимаются на орбитали с большой энергией, потом дружно оттуда падают и при этом излучают. Но я не понимаю, что я должен сделать на практике, чтобы накачать лазер и как оно будет связано с переходом электронов между энергетическими уровнями.
Вот пусть у меня есть баллон аргона. Что еще мне нужно, чтобы собрать лазер? Что будет происходить при его накачке? Как посчитать необходимые параметры?
Как провернуть то же самое, если вместо баллона аргона у меня кристалл рубина? Из того, что я понял из википедии, его надо тупо поместить между двумя зеркалами, одно из которых полупрозрачное (полу - это, кстати, насколько?), и посветить на него лампочкой. Но в чем назначение этих зеркал? Как будет соотноситься мощность лампы и лазера?
>> No.141987 Reply
>>141930
бамп
>> No.141988 Reply
>>140769
Если ты изучаешь школьный курс, то ВООБЩЕ не важно по какому учебнику учить. В школьном курсе все равно ничего из того, что тебе понадобиться за гранью ЕГЭ. Учи по тому, что есть. Как пойдешь дальше, так и список учебников появится.
>> No.142012 Reply
>>141930
Ну, я книжку какую-то нашёл: http://libgen.biz/book/index.php?md5=621e83eb32e41f986e967e7854d9a938
Не знаю, насколько полезную, но, судя по всему, довольно подробную.
>> No.142052 Reply
>>142012
Кошмарная книжка. Формулы, таблицы, формулы. Для того, чтобы расчитывать всякое, это и нужно, но чтобы понять смысл происходящего, совершенно не подходит.
Я нашел хорошую лекцию, в которой поясняется за зеркала:
http://www.youtube.com/watch?v=-cLSnuXGC_U
И в принципе, мне теперь понятна работа лазера с оптической накачкой.
А вот с газовыми лазерами, я до сих пор не понимаю принципа накачки. Почему разряд, проходящий через газ, создает инверсную населенность? Это свойство плазмы? Типа, электроны, вылетевшие из атома, оседают на другой атом и садятся на высокие энергетические уровни?
Кроме того, возникает совсем уж фундаментальный вопрос. Как, черт возьми, работает зеркало?
>> No.142071 Reply
>>142052
Зеркало - всего лишь материал в высокой степени не пропускающий/не поглощающий/отражающий фотоны.
>> No.142073 Reply
>>142071
Его, вероятно, интересует отражение от поверхности металла.
>> No.142136 Reply
Почему при равномерном движении по окружности (например колесо равномерно катится по земле без проскальзываний) модуль скорости точек на поверхности колеса равен модулю скорости центра? Как это доказывается? Почему длина пути точки на колесе относительно центра равна длине пути центра относительно земли?
Допустим есть такая задача: Шарик массы m движется по окружности на ните длинны l под углом альфа к вертикали. Найти скорость шарика. Тогда мы расписываем силу натяжения нити по вертикали и горизонтали через cos и sin альфы. Приравниваем составляющую силы натяжения нити по вертикали к силе тяжести, находим всю силу натяжения, находим её составляющую по горизонтали, находим ускорение и оттуда скорость. Так? Но чем отличается подобная ситуация ирл? Ведь если я подвешу шарик, он просто начнет какое-то время качаться, пока нить не станет вертикальна, а не вращаться по окружности. Что не так и почему? Что вращает шарик в задаче?
>> No.142137 Reply
>>142136
Ты прочитал про теорему Шаля?
>> No.142149 Reply
File: d17d4dd0e1674260a3316e606e65f8bdf2eb828d.jpg
Jpg, 737.89 KB, 700×1000 - Click the image to expand
edit Find source with google Find source with iqdb
d17d4dd0e1674260a3316e606e65f8bdf2eb828d.jpg
>>142136
> модуль скорости точек на поверхности колеса равен модулю скорости центра?
Тебе не кажется, что иначе бы колесо развалилось? Твои вопросы, из-за того, что ты путаешь скорость относительно стороннего наблюдателя и скорость относительно центра колеса. Ну скорее всего из-за этого. Алсо, строгого доказательства не помню, чтобы где-нибудь видел.
> Почему длина пути точки на колесе относительно центра равна длине пути центра относительно земли?
Насколько я помню, такое выполняется, только когда идет движение без проскальзывания.
> Что вращает шарик в задаче?
В задаче сказано, что параметры таковы, что равнодействующая всех горизонтальных сил равно нулю. С чего ему падать?
> Шарик массы m движется по окружности
Почему ИРЛ так не происходит? Это просто. Потому что не надо тащить физику в ИРЛ, также как в аниме. Задача составлена с целью донесения до тебя инфы и не более. Это учебная задача. В ней много допущений.
>> No.142161 Reply
>>142136
> Но чем отличается подобная ситуация ирл? Ведь если я подвешу шарик, он просто начнет какое-то время качаться, пока нить не станет вертикальна, а не вращаться по окружности.
Возьми шарик на нити, раскрути его по горизонтали (двигая нить) и подвесь. Крутится? Ну вот тебе и начальные условия.
>> No.142252 Reply
>>142137
Нагуглил и ничего не понял. Я закончил 10 класс и в движения в геометрии ещё не могу. А уж применять эту штуку к физике - тем более.
>>142149
Нет, не кажется. Оно бы точно развалилось, если бы скорости точек на поверхности отличались по модулю друг от друга. А если они отличаются от модуля скорости центра, то все вроде бы в порядке.
> Насколько я помню, такое выполняется, только когда идет движение без проскальзывания.
А почему такое выполняется?
>> No.142257 Reply
>>142252
Реши задачу с проскальзыванием.
>> No.142343 Reply
Как в класических моделях обстоит вопрос гравитации через порталы и червоточины? Насколько я понимаю, в момент открытия в них полетят гравитационные поля, вместе с материем, излучением и всем прочим, т.е. на каждом конце червоточины получим суммарное притяжение того, что на другом её конце. Если червоточину откроют между двумя планетами, то эта червоточина внезапно станет супермассивной дыркой с массой в целую планету, и это будет происходить на обоих концах. Или я где-то ошибаюсь?

помимо этого, слышал что по нормальному червоточина будет размером где-то с атом, и чтобы её разработать пошире, понадобится НЕХ вроде отрицательной энергии. Но для передачи фотонов такой дырочки должно хватить, так? Т.е. - сверхсветовая связь возможна?
>> No.142349 Reply
>>142343
> Т.е. - сверхсветовая связь возможна?
имплаинг червоточины доказаны и возможны
>> No.142401 Reply
>>142349
Ну, они какбе математически доказаны. Но все равно, как теории на их счет обращаются с этим вопросом?
>> No.142492 Reply
>>142257
Какую задачу с просальзованием? О чем ты?
>> No.142493 Reply
>>142401
Ничему не противоречат != доказаны.
>> No.142520 Reply
>>142493
отсутствие доказательств != доказательство отсутствия
>> No.142537 Reply
>>137453
Никаких особенных проблем на уровне мат. аппарата. Все его конструкции, в принципе, обобщаются на такую ситуацию. А вот как ты будешь ИНТЕРПРЕТИРОВАТЬ эту математику физически, если время у тебя даже локально не образует упорядоченного множества - это уже очень отдельный вопрос, и собственно, в этом и основная проблема.
>> No.142538 Reply
>>142537
Любое множество может быть вполне упорядочено. Любое. Абсолютно.
>> No.142540 Reply
>>142538
Да вот только редко какой порядок ты можешь выразить конечным выражением.
>> No.142567 Reply
>>142538
Слышал звон, да не знаю, где он. Во-первых, как ты планируешь работать с упорядочением, про которое ты не знаешь ничего, кроме того, что оно есть? Ты не сможешь даже сравнить между собой никакие два конкретных элемента этого множества, потому что аксиома выбора даёт только принципиально неконструктивные ответы. Во-вторых, далеко не на всяком множестве можно ввести упорядочение так, чтобы оно было согласовано с топологической и алгебраической структурой, которая там уже имеется. К примеру, в R^2 это невозможно.
>> No.142568 Reply
>>142540
Смотря как понимать "конечным". Символом < я могу упорядочить любое множество. Более того, этот порядок будет полным: в любом непустом подмножестве найдётся наименьший элемент.
>> No.142570 Reply
>>142567
Работать с этим порядком нужно трансфинитной индукцией, как водится.
Идея, что время не образует упорядоченного множества, неверна. Любое множество можно упорядочить. Большего я и не утверждал.
>> No.142571 Reply
>>142570
> Любое множество можно упорядочить.
От аксиоматики зависит. >>142567-кун правильно говорит: это упорядочение в общем случае бесполезно.
>> No.142572 Reply
>>142571
Наверняка информации хватит, чтобы работать с этим странным временем как с более приличным привычно упорядоченным временем - не зря же его назвали временем, в конце концов. Возможно, можно будет делать утверждения вида "если частица прошла через все моменты времени, меньшие t, то частица прошла и через момент t", что, при некоторой фантазии, позволяет получить много информации о частице.
>> No.142590 Reply
>>137425
Физику вообще не знаю, с чего начать, анон? Школу закончил четыре года назад.
>> No.142600 Reply
>>142590
Ландсберг.
>> No.142611 Reply
>>142572
>>142570
> не зря же его назвали временем
Исключительно из-за знака минус в сигнатуре метрического тензора. В противном случае это были бы обыкновенные пространственные координаты.
> Работать с этим порядком нужно трансфинитной индукцией, как водится.
Во-первых, трансфинитная индукция не всесильна. Далеко не все верные утверждения можно доказать с её помощью.
Во-вторых, это не так просто, как тебе кажется. Теорема Цермело не даёт тебе никакой информации о свойствах того отношения упорядочения, существование которого она утверждает. Ты не знаешь даже, какой именно элемент является минимальным во всём множестве. То есть даже базу индукции ты не докажешь - это ничуть не проще, чем напрямую доказывать интересующее тебя утверждение для всех элементов множества. Помню, как-то я решил, пользуясь трансфинитной индукцией, доказать, что все непрерывные отображения измеримы. Теорема Цермело в её обычном виде оказалась совершенно бесполезной в этом деле. Мне пришлось разбирать её доказательство на запчасти и конструировать заново, вручную согласовывая со структурой сигма-алгебры. То же самое придётся делать и тебе, для каждой доказываемой теоремы изобретая отдельный способ упорядочения, в то время как для времени такое упорядочение должно быть единым и априори заданным.
В-третьих, далеко не все интересующие физиков утверждения имеют вид "для любого момента времени t выполнено P(t)". Допустим, у тебя есть два конкретных события, и ты хочешь узнать характер их причинной взаимосвязи. Если их разделяет пространственноподобный интервал - понятно, прямой причинно-следственной связи нет. А если интервал времениподобен? Какое из них является причиной, а какое следствием? Ответить на этот вопрос можно, только сравнив временные координаты этих событий. Чего ты сделать не можешь, потому что твоё отношение упорядочения неконструктивно.

Впрочем, просто упорядочить (хоть и не "вполне") R^2 как множество можно и конструктивно, но...
> Идея, что время не образует упорядоченного множества, неверна. Любое множество можно упорядочить.
Время вообще не образует "просто множества". Время образует, если угодно, топологическое векторное пространство над полем R. Да, обычно они моделируются теоретико-множественно, но вообще говоря, это объект из другой категории. Вещественное векторное пространство, наделённое стандартной топологией, упорядочить НЕЛЬЗЯ, если его размерность больше единицы. Это тривиальный факт, проистекающий из того, что при удалении нуля (да и вообще любой точки) такое пространство по-прежнему остаётся связным, в то время как должно разбиваться как минимум на две компоненты связности - "больше нуля" и "меньше нуля".
>> No.142727 Reply
File: 1.1.jpg
Jpg, 574.27 KB, 2592×1552
edit Find source with google Find source with iqdb
1.1.jpg
File: 1.2.jpg
Jpg, 583.48 KB, 1552×2592
edit Find source with google Find source with iqdb
1.2.jpg

Сап, анон, помоги мне.
В ответах к первой задаче написано, что выигрыш в силе в 2 раза. Чому? Человек же будет просто поднимать свой вес, как он есть. дяьвол, какой же я бака
>> No.142729 Reply
>>142727
Потому что две точки приложения силы натяжения веревки. А гравитация - одна. При условии равенства сил получаем, что сила натяжения в два раза меньше силы тяжести. В руке ты держишь только один конец, поэтому и прикладываешь силу в два раза меньше. Другая половина приходится на крепление.
>> No.142748 Reply
>>142729
На человека в коробке действует вниз сила тяжести G. В месте крепления коробки к веревке вверх действует сила натяжения нити Т. Она "перебрасывается" через блок, и точно такая же сила Т действует вверх там, где человек берется за веревочку. Значит, что бы двигать коробку он должен тянуть с силой F=T=G. Не понимат.
>> No.142749 Reply
File: хуле_я_за_тебя_все_делаю.png
Png, 5.58 KB, 291×357 - Click the image to expand
edit Find source with google Find source with iqdb
хуле_я_за_тебя_все_делаю.png
>>142748
Не понимаешь - нарисуй.
>> No.142751 Reply
File: T_T.jpg
Jpg, 111.52 KB, 1620×1215 - Click the image to expand
edit Find source with google Find source with iqdb
T_T.jpg
>>142749
А я рисовал... И так, как ты нарисовал, все ок. Но ведь коробка не повешена на блок с помощью нити, закрепленной на коробке. Там один конец свободный же. Неподвижный блок не дает выигрыш в силе, только изменяет направление в силе..
>> No.142752 Reply
>>142751
В каком месте он свободный? Он находится в руке человека, который сидит на коробке. То есть в итоге точно так же прикреплен к ней.
>> No.142753 Reply
>>142752
Ну вроде и так, а вроде и не так. Ну вот на "T_T" смотри. Там точно все правильно. Вот я тяну коробку с F=T=G, приподнимаю её на пару см над семлей, подхожу к ней и сажусь на неё(допустим я ну очень очень легонький). И вот я сижу на коробке, которую как тянул так и тяну с силой F=G. В какой момент и почему она стане G\2?
>> No.142754 Reply
File: висельник.png
Png, 13.21 KB, 634×561 - Click the image to expand
edit Find source with google Find source with iqdb
висельник.png
>>142753
А ты рассмотри силы, действующие на тебя. Потом вспомни, что сила действия равна силе противодействия, и ты на коробку действуешь с той же силой N, что и она на тебя. Если масса коробки пренебрежимо мала по сравнению с твоей, N будет равно T = g/2.
>> No.142755 Reply
File: ^-^.jpg
Jpg, 547.59 KB, 3768×4000 - Click the image to expand
edit Find source with google Find source with iqdb
^-^.jpg
>>142754
Так?
>> No.142756 Reply
>>142755
Именно так.
>> No.142757 Reply
>>142756
пасиб :3
>> No.142787 Reply
File: Wm9v0AnZnQE.jpg
Jpg, 34.43 KB, 600×436 - Click the image to expand
edit Find source with google Find source with iqdb
Wm9v0AnZnQE.jpg
Сап, доброанон. Мне указали на некоторые разделы математики, овладение которыми обязательно при изучении физических дисциплин. В список вошли, в том числе: линейная алгебра, матан, теорвер, мат. статистика, геометрия (школьный курс), функан, дифференциальная геометрия и топология. Какие книги вы посоветуете, чтобы качественно проникнуться этими темами? Какие разделы, с вашей точки зрения, были нагло упущены? Насколько глубоко физику-теоретику/программисту (моделирование физических процессов) необходимо разбираться в математических концепциях?
Для того, чтоб не терять время, и с целью освежения памяти, читаю Шеня и Давидовича. Годные авторы?
>> No.142792 Reply
>>142787

Я бы сказал, что упущены диффуры и комплексный анализ, причём это очень важные упущения. Хорошо бы также немного знать теорию групп. А вот с такой уж необходимостью школьного курса геометрии я бы поспорил. Далее идут уравнения в частных производных и группы и алгебры Ли.
> Насколько глубоко физику-теоретику необходимо разбираться в математических концепциях?
Ответ на этот вопрос зависит от того, кого считать физиком-теоретиком. Атья говорил про Виттена "Although he is definitely a physicist (as his list of publications clearly shows) his command of mathematics is rivaled by few mathematicians", и это может быть сказано про очень многих физиков мирового уровня. По-настоящему крупные физики наших дней знают математику очень и очень хорошо. Если же брать в качестве ориентира среднего человека, защитившего диплом по специальности "физика", то он может не знать вообще ничего, кроме того, как брать интегралы, и жить себе припеваючи.
> Какие книги вы посоветуете, чтобы качественно проникнуться этими темами?
линейная алгебра - Винберг("Алгебра"), Кострикин-Манин.
матан - Зорич, Львовский
комплан - Шабат, может быть, Картан("элементарная теория аналитических функций")
функан - Хелемский, Кириллов-Гвишиани, Колмогоров-Фомин
дифференциальная геометрия и топология - Милнор("Теория Морса", "Дифференциальная топология"), часть курса матана Львовского, тут можно назвать много учебников.
диффуры - Понтрягин, Арнольд
уравнения в частных производных - Шубин, Арнольд.
Группы и алгебры Ли - Серр, Винберг-Онищик.
Насчёт теорвера и матстата я не советчик, не припомню ни одного учебника этих дисциплин, который бы мне нравился.
> читаю Шеня и Давидовича. Годные авторы?
Под Давидовичем ты имеешь в виду "матан в 57ой школе"? Хорошая книжка.
Что до Шеня, то он написал много книг. Некоторые из них для физика - трата времени, некоторые - нет.
>> No.142794 Reply
File: 14124167125471.jpg
Jpg, 30.64 KB, 550×391 - Click the image to expand
edit Find source with google Find source with iqdb
14124167125471.jpg
>>142792
> кого считать физиком-теоретиком
Например, Фейнмана, Ландау, Эйнштейна как же меня бесит неправильная передача его фамилии на русском. Фейнман, к примеру, пишет в своих лекциях, что он не помнит всех математических выкладок, которые относятся к физике, но понимая основные принципы, способен выводить законы самостоятельно. Эйнштейн, и это хорошо известный факт из его биографии, долгое время ничего не мог сделать с длинными уравнениями ОТО, пока не познакомился с Леви-Чивитой и его новым математическим аппаратом - тензорным исчислением. Значит, некоторые пробелы в знании математики присутствовали. Ландау не читал статьи самостоятельно. Это дело он поручал своим аспирантам и сотрудникам, которые позже пересказывали содержание. Если статья его интересовала, он самостоятельно приходил к тем же выводам, окольными путями.
> комплан - Шабат, может быть, Картан("элементарная теория аналитических функций")
Если быть уж совсем точным, которая из них лучше? У меня, в данный момент, на руках есть Шабат "Методы теории функции комплексного переменного".
> Некоторые из них для физика - трата времени, некоторые - нет.
Будь добр, конкретней.

И ещё, меня беспокоит пара вопросов. Насколько хорошо физику необходимо овладеть логикой? Какими конкретно разделами? Стоит ли углубиться в математические разделы, которые были указаны выше, пользуясь монографиями? Есть доступ к библиотеке одного профессора, который МФТИ закончил, не хочу упустить шанс. Тем более, он мне дарит эти самые книги.
>> No.142795 Reply
>>142794
> Например, Фейнмана, Ландау, Эйнштейна
Неоднократно и от знающих людей я слышал, что если бы Фейнмана по-человечески учили математике, а не натаскивали брать производные, то у нас сейчас была бы строгая теория континуального интеграла. Так что есть мнение, что Фейнману матан надо было подтянуть. Насчёт Эйнштейна ты сам всё сказал: при помощи существовавшей на то время математики он работать не мог, ему нужны были новейшие методы. Он знал всю математику, и то ему не хватало. Сделай выводы.
> Если быть уж совсем точным, которая из них лучше?
Обе лучше. Шабат существенно полнее, Картан существенно понятнее(по-моему).
> "Методы теории функции комплексного переменного".
Я скорее имел в виду "введение в комплексный анализ". Насколько я помню, "методы" содержат меньше материала, и изложение начальных сведений во "введении" попроще.
> Насколько хорошо физику необходимо овладеть логикой?
Математической? Нинасколько. Собственно, под тратой времени для физика я подразумевал книжки Шеня по математической логике.
> Стоит ли углубиться в математические разделы, которые были указаны выше, пользуясь монографиями?
Не понимаю вопроса. Монография есть книга, посвящённая одной конкретной теме и рассматривающая эту тему глубоко. Например, "топологические методы алгебраической геометрии" Хирцебруха - монография. Стоит ли тебе читать монографии? Хорошие монографии - стоит, плохие - лучше не надо. Думаю, ты использовал слово "монография" своим особым способом.
> Тем более, он мне дарит эти самые книги.
Пользуйся, если дарит. Бумажная книга всегда удобнее, да и выпускник МФТИ хуйни не посоветует, думаю. Собственно, я просто назвал учебники, которые изучал сам, и которые мне понравились, список может быть скорректирован в любую сторону.
>> No.142797 Reply
File: S_ccrRKNN_U.jpg
Jpg, 54.37 KB, 604×472 - Click the image to expand
edit Find source with google Find source with iqdb
S_ccrRKNN_U.jpg
>>142795
> Думаю, ты использовал слово "монография" своим особым способом.
   Нет, именно так, как ты описал. Книга, к примеру, по тензорному анализу - монография, т.к. тензорный анализ - раздел дифференциальной геометрии. Налицо: одна проблема, глубокое рассмотрение.

Но, в целом, спасибо за ответы.
>> No.142799 Reply
>>142792
> матан - Зорич, Львовский
«Дифференциальное исчисление. Дифференциальные формы» Картана стоит ещё добавить.
>> No.142800 Reply
>>142799
На вкус и цвет. По-моему, это не самая лучшая книга, и уж точно не самая подходящая для физика. Предлагаю в качестве альтернативы "анализ" Шварца.
Также нужно добавить что-то по вариационному исчислению. Тут засада, т.к. все книги этой теме, которые я держал в руках, были либо неполны(Гельфанд), либо ужасно написаны. Есть у кого-нибудь идеи?
>> No.142802 Reply
>>142800
> На вкус и цвет. По-моему, это не самая лучшая книга, и уж точно не самая подходящая для физика.
Не знаю, не знаю. Язык там, по-моему, лучше, чем у Шварца.
Кроме того, я бы не назвал Львовского подходящим для физиков.
>> No.142811 Reply
File: 14210159798900.jpg
Jpg, 181.48 KB, 1280×960 - Click the image to expand
edit Find source with google Find source with iqdb
14210159798900.jpg
>>142792
> Насчёт теорвера и матстата я не советчик, не припомню ни одного учебника этих дисциплин, который бы мне нравился.
Так что, ребят, кто-нибудь посоветует годные книги по этим двум разделам? Мне, однажды, Колемаева рекомендовали. И стоит ли брать один учебник, в котором излагается теорвер, а после него матстат, как это принято в университетской практике? Или, всё же, отдать предпочтение двум книгам?
>> No.142812 Reply
>>142811
Who is Kolemaeff?
>> No.142813 Reply
>>142812
ruwiki://Колемаев,_Владимир_Алексеевич
Очевидно, что он - совсем не физик.
>> No.142815 Reply
Товарищи, посоветуйте видеолекции по физике.
>> No.142828 Reply
File: IMAG1486.jpg
Jpg, 810.87 KB, 2592×1552
edit Find source with google Find source with iqdb
IMAG1486.jpg
File: IMAG1485.jpg
Jpg, 993.45 KB, 2592×1552
edit Find source with google Find source with iqdb
IMAG1485.jpg

Привет, Доброчан, это опять я. Нужны 2 задачи пикрил.
Первую я вроде решил, но недоволен. В Ландсберге было написано: "... когда работа некоторой силы F отрицательна, работа, совершаемая какой-то другой силой против силы F, будет положительна". Если использовать это и написать, что Mgh=Fl (Где h - высота клина, а l - длина), то ответы сойдутся. Но:
1. Эта фраза была написана при рассмотрении случая, когда силы коллинеарны (был рассмотрен груз, падающий вниз и прикрепленный к потолку пружиной), в этой же задаче силы перпендикулярны. Да и нельзя сказать, что сила F делает работу против силы тяжести поршня, ведь на клин действует только вес P поршня.
2.Я вообще впервые слышу эту фразу о работе.
3.Вообще, если рассматривать эту задачу без трения, то на поршень действуют только сила тяжести и нормальная реакция опоры. Тогда их равнодействующая совсем не ноль. Чому клин стоит? Или учитывать то, что поршень как-то там сверху закреплен и потому, в сторону двигаться не может?
Я так же попытался решить задачу через динамику: на поршень действует сила тяжести, её составляющая, перпендикулярная верхней грани клина, есть вес, действующий на клин. Значит на клин действуют вес поршя и сила F. Значит, что бы вогнать клин равномерно под поршень, горизонтальная составляющая веса должна быть равна F. Но если все так сделать, получается совсем другой ответ. Чому?
А как делать вторую через работу я совсем не знаю. Да и если не через работу. Разве что так:
Допустим, винт сделает один шаг, давя с силой P, тогда работа, совершенная винтом, будет равна Ph(где P=10^4H, h=5мм). В то же время за один шаг винта рукоятка сделает полный оборот, значит работа силы, приложенной к ней, будет равна 2πr*F(где r=40см, F - искомая сила). Тогда, раз машина движется равномерно и не деформируется, 2πrF=Ph=>F=Ph/(2πr). Верно?%% ответа к этой задаче в учебнике нет%%
>> No.142829 Reply
>>142828
> Эта фраза была написана при рассмотрении случая, когда силы коллинеарны (был рассмотрен груз, падающий вниз и прикрепленный к потолку пружиной), в этой же задаче силы перпендикулярны. Да и нельзя сказать, что сила F делает работу против силы тяжести поршня, ведь на клин действует только вес P поршня.
Закон сохранения энергии. Сила F совершает работу (так как осуществляет перемещение клина), единственное место, куда она может деваться — потенциальная энергия поршня.
> Чому клин стоит?
Его же вдвигают. И да, поршень может двигаться лишь вертикально.
> Значит на клин действуют вес поршя и сила F.
Ещё реакция опоры под клином.
> А как делать вторую через работу я совсем не знаю. Да и если не через работу. Разве что так:
Выглядит вполне разумно.
>> No.142867 Reply
>>142829
Про законы сохранения в учебнике ещё ничего не сказано, рассматривать можно только работы. Вообще не люблю законы сохранения энергий. Поршень и клин - замкнутая система (или незамкнутая из-за Земли?). Значит приращение потенциальной энергии равно работе неконсервативных сил => Mgh=Fl?
А все же, как сделать без работ и без энергий? Через динамику. На клин действует вес поршня, сила F и нормальная реакция опоры. Что бы клин двигался, сумма проекций сил на горизонталь должна равняться нулю => F=Px. Но если выражать сначала вес, потом его горизонтальную составляющую, получается совсем другой ответ.
> Выглядит вполне разумно.
А по-моему нет. Опять же, впервые слышу, что если машина не ускоряется и не деформируется, то работа, совершенная над ней, равна работе, совершаемой ею. Да и что принимать за машину (какие части машины), что понимать под работой над машиной. Мутная и неясная формулировка. Да и
> > Допустим, винт сделает один шаг, давя с силой P
А если под винтом абсолютно твердая дощечка? Нижний конец никуда не сдвинется и работа совершенна не будет, но все равно будет давление. Как тогда решить? Или ещё с каких других позиций?
>> No.142868 Reply
>>142867
> Вообще не люблю законы сохранения
Зря, ньютоновская механика как раз из них ну и принципа наименьшего действия и выводится. И многие задачи через энергию решать проще.
> Поршень и клин - замкнутая система (или незамкнутая из-за Земли?).
Считай, что замкнутая — поле постоянно.
> Значит приращение потенциальной энергии равно работе консервативных сил => Mgh=Fl?
Именно так.
> Но если выражать сначала вес, потом его горизонтальную составляющую, получается совсем другой ответ.
Я не знаю, как именно ты решал же.
> Опять же, впервые слышу, что если машина не ускоряется и не деформируется, то работа, совершенная над ней, равна работе, совершаемой ею.
Опять же, закон сохранения энергии.
> что понимать под работой над машиной
У работы в механике достаточно узкое определение.
> А если под винтом абсолютно твердая дощечка?.
Тогда полагаешь смещение бесконечно малым и получаешь тот же ответ. Т.к если мы поворачиваем винт на угол α, то αrF = P hα/(2π) => F = Ph/(2πr).
>> No.142893 Reply
File: jpg.jpg
Jpg, 544.90 KB, 5000×4008 - Click the image to expand
edit Find source with google Find source with iqdb
jpg.jpg
>>142868
Принцип наименьшего действия? Что? Я все делаю через 3 закона ньютона и ,может, импульсы. А энергии использую только если без них никак или очень трудно.
> поле постоянно.
Какое поле? Поле силы притяжения земли? Потенциальное поле? Не знаю таких штук ещё..
> как именно ты решал же.
Пикрил.
> Тогда полагаешь смещение бесконечно малым
Ок, это понял.
>> No.142900 Reply
>>142893
> Принцип наименьшего действия? Что?
Не надо так резко реагировать на спойлеры. О принципе наименьшего действия можешь не беспокоиться — это лагранжева механика, уже за пределами школьной физики.
> А энергии использую только если без них никак или очень трудно.
Лучше использовать их чаще.
> Какое поле? Поле силы притяжения земли? Потенциальное поле? Не знаю таких штук ещё..
В данном случае — поле силы тяжести.
> Пикрил.
Как-то странно у тебя силы разложены... Потом, наверное, посмотрю.

Да, советую добавить к Лансбергу ещё и фейнмановские лекции по физике.
>> No.142910 Reply
File: 14368170188860.jpg
Jpg, 20.33 KB, 679×104 - Click the image to expand
edit Find source with google Find source with iqdb
14368170188860.jpg
Доброанон, ответь на вопрос. Пикрелейтед, задание 2.

В предшествующем заданиям параграфе, речь шла о взаимном притяжении и отталкивании молекул. На элементарных примерах объяснялось, что поверхности, покрытые маслом или другими жирами, будут отталкивать воду - гидрофобные (конкретно этот термин не применялся, но он подразумевается).

Я незамедлительно провёл эксперимент, смочив половину листа бумаги водой, а вторую половину - подсолнечным маслом. Теперь, внимание, они - слиплись. Конечно, я понимаю, что это "слипание" не тождественно тому, которое описано в 1 задании пикрелейтеда.

Смоченные в воде кусочки бумаги слипаются. После высыхания, если попытаться их разорвать, мы получим более тонкий вариант одной части листка и второй листок с шероховатой поверхностью. Получается, благодаря воде, волокна целлюлозы сблизились на расстояние, которое подразумевает действие силы притяжения между одинаковыми молекулами вещества.

Какова роль воды в этом случае? Почему вообще одни вещества подвержены смачиванию, а другие - нет? Как происходит взаимодействие молекул воды и целлюлозы? Они "выталкивают" целлюлозу наверх, где молекулы могут соприкоснуться с родственным им вторым листом бумаги? Вода нарушает кристаллическую решетку вещества или деформирует её? Заполняет пустые промежутки кристаллической решетки?

Возвращаясь ко второму заданию, замечу, что с моей точки зрения, вода высохнет быстрей масла и в конечном итоге бумага слипнется.

Так почему листок, обработанный маслом, соприкоснулся со смоченным в воде? Масло оказалось плохим? Или молекулы целлюлозы взаимно притянулись? Почему жиры не растворяются в воде, а образуют эмульсию?
>> No.143047 Reply
Доброанон, ответь на вопрос.

"На рычаге уравновешены две железные гирьки массами 100 и 200 г. Нарушится ли равновесие рычага, если гирьки погрузить в воду?"

Моя точка зрения такова, что равновесие сохранится. В решении написано обратное: "В воде равновесие нарушится, т.к. тела разной массы и одинаковой плотности имеют разный объем и выталкивающая сила действующая на них различна. ".

Я не согласен с этим ответом. Согласно правилу рычагов: F1/F2=l1/l2.
Когда мы погружаем нашу установку в воду, плечи, очевидно, не меняют своей длины. В таком случае, для соблюдения равновесия, силы, выталкивающие тела из воды должны соблюдать пропорцию. Первая гирька, массой 100 г., обладает весом в 1Н (приблизительно). Соответственно, вторая - 2Н. Пропорция = 1/2. Погружая гирьки в воду, на них начинает действовать сила Архимеда. Легко посчитать, что и здесь пропорция сохраняется и равна 1/2. Значит, равновесие не нарушится. Кто неправ?
>> No.143091 Reply
>>143047
Думаю, я с тобой соглашусь, так как вне воды силы равны ρ[г]gV[i], а в воде (при полном погружении) — (ρ[г] - ρ[в])gV[i], и в итоге значение имеет лишь отношение объёмов.
В случае гирь разной плотности, равновесие действительно было бы нарушено.
>> No.143132 Reply
>>142900
Ну как, посмотрел силы?


Алсо, кто может дать годные сайти для подготовки к ЕГЭ и олимпиадам? Желательно с вариантами и все такое.
Особенно интересуют варианты олимпиады Росатом для 10 класса этого года.
>> No.143256 Reply
File: IMAG1492.jpg
Jpg, 656.47 KB, 2592×1552
edit Find source with google Find source with iqdb
IMAG1492.jpg
File: IMAG1491.jpg
Jpg, 543.24 KB, 1552×2592
edit Find source with google Find source with iqdb
IMAG1491.jpg

2 пика. Ответ в учебнике - 36000км. А у меня нет. Почему?
>> No.143272 Reply
>> No.143281 Reply
Прочитал про неинерциальные системы отсчета, теперь совсем запутался..
Допустим есть диск, вращающейся в горизонтальной плоскости а на нем тело. Если диск будет вращаться слишком быстро, то тело с него слетит. Почему? Если рассматривать относительно диска, то нужно учитывать силу инерции и все ок - возрастает скорость вращения => возрастает нормальное ускорение => возрастает сила инерции и когда она превосходит силу трения покоя, тело сбрасывается. А если рассматривать относительно Земли (примем её как инерциальную)? Какая сила скинет тело с диска?
>> No.143282 Reply
>>143281
> Какая сила скинет тело с диска?
Не сила, инерция. Напомню, что без действия сил тело движется равномерно и прямолинейно, и нужно приложить силу чтобы изменить импульс тела.

PS: Фейнмана читаешь?
>> No.143290 Reply
>>143282
Нет, Ландсберга. Феймана тоже хочу потом попробовать.
Ну так, на тело на диске будут действовать: сила тяжести, нормальная реакция опоры и сила трения убегающего из под тела диска. Сила тяжести и нормальная реакция никак на движение не повлияют. А сила трения диск-тело будет действовать по касательной к окружности и направлена в сторону вращения. Но она не зависит от скорости диска.
Так, зря я об этом так много думаю, теперь ещё вопрос. На тело на диске действует сила трения диск-тело и сила трения покоя тела. Как найти первую?
>> No.143291 Reply
>>143290
Ох черт, ещё вопрос. Если тело лежит на вращающемся диске, оно имеет центростремительное ускорение. Чем оно обусловлено? Силы трения направлены по касательной, а силы тяжести и нормальной реакции опоры в вертикальной плоскости. Что действует в центр? (Относительно ИСО)
>> No.143301 Reply
File: DIblbEgQj4I.jpg
Jpg, 48.99 KB, 460×604 - Click the image to expand
edit Find source with google Find source with iqdb
DIblbEgQj4I.jpg
Вопрос, доброаноныаноны. Размышлял на досуге о конвекции и задался вполне логичным вопросом: существует ли конвекция в плазме? Погуглил - ответ был утвердительным. Теперь суть моего вопроса. Куда направленны конвекционные потоки, например, внутри Солнца и почему? Ведь ядро более горячее, чем поверхность. Более разогретое вещество == менее плотное, ввиду этого, под действием силы Архимеда менее плотное "плавает" в более плотном, температура должна быть выше снаружи, по аналогии с обычной комнатой, в которой холодный воздух внизу, а горячий - вверху. Но с другой стороны, если принять во внимание эквивалентность энергии и массы (не уверен, насколько это допущение законно),тела, обладающие большей энергией, обладают большей массой. У более горячих частиц кинетическая энергия выше. Значит, они более массивные. В таком случае, на тело большей массы должна действовать огромная сила тяжести, направленная к ядру звезды, в данном случае, Солнца. Архимедова сила скомпенсирует действие силы тяжести?
Не гоните меня ссаными тряпками, пожалуйста. Я действительно хочу разобраться в данном вопросе.
>> No.143302 Reply
>>143291
Ставишь телегу впереди лошади. Тело движется не прямолинейно - значит, его скорость меняется. Если скорость меняется, то у тела есть ускорение. Если у тела есть ускорение, то на него действует сила. Тебе надо понять, почему тело движется не прямолинейно. Если поднять его с диска, то будет ли тело двигаться по окружности? Очевидно, что нет. Значит, сила спрятана где-то во взаимодействии тела и диска, которое появляется, когда тело стоит на диске. Как конкретно взаимодействуют два куска вещества, сказать сложно, нужно изучать всякие натяжения-растяжения-трения и так далее.
>> No.143303 Reply
>>143301
Там ещё и электричество надо учитывать, например.
ruwiki://Протуберанец
>> No.143309 Reply
>>143303
Это всё замечательно, но ответа на свой вопрос я не получил. Вопрос был: "Куда направленны конвекционные потоки, например, внутри Солнца и почему?". Естественно, я понимаю, что моё предположение - не окончательная инстанция и может в корне отличаться от действительности.
>> No.143323 Reply
>>143309
Они в обе стороны направлены. Вообще, когда низ нагревается, а верх охлаждается, получается вот это:
ruwiki://Ячейки_Бенара
В случае Солнца - думаю, то же самое, но в сферическом слое вокруг ядра.
>> No.143334 Reply
Доброчан, посоветуй годные задачники к учебникам по школьной физике.

Также, хотелось бы услышать мнение местного анона относительно учебников физики Пёрышкина за 7-9 классы и Мякишева за 10-11. Годные?
>> No.143335 Reply
>>143334
Старый(очень старый) пёрышкин годный. Новый хуйня. А тебе зачем, вести в школе физику будешь?
>> No.143336 Reply
>>143335
Нет. Украинское образование настолько ущербно, что читать можно только русскоязычные учебники. реальная история я не тролль Мне необходимо освежить память после школы и закрепить полученные знания. Так что относительно Мякишева и задачников?
>> No.143338 Reply
>>143336
Гугли профильные учебники Мякишева (их 5 всего: механика, мкт и термодинамика, электродинамика, колебания, оптика и квантовая физика). Еще годный учебник у Яворского в двух томах, но для чтения нужно иметь уже какие-то базовые знания. Задачи можешь порешать из какого-нибудь Рымкевича, хоть он и унылый. Еще есть старое пособие Мясникова с разобранными задачами, мне оно очень понравилось.
>> No.143344 Reply
>>143336
Если с математикой дружишь - возьми лучше Сивухина.
>> No.143348 Reply
>>143344
Что из себя представляет Детлаф? Чем он отличается, например, от того же Фейнмана?
  
Почему некоторые так принижают Л.-Л.?

Задачник Воробьева-Зубкова хорош?

Что за учебник у Ландсберга?

Какие темы из школьного курса геометрии наиболее часто фигурируют в физике? (помимо очевидных: тригонометрии, геометрии треугольника и других плоских/объемных фигур, векторного анализа) Какие учебники позволяют вдоволь проникнуться именно нужными темами? Не слишком математизированные, физики ведь не ставят перед собой цель решать олимпиадные задачи по геометрии.
>> No.143351 Reply
>>143348
Элементарный учебник физики. Под ред. Г.С. Ландсберга. Мне нравится. Правда я ещё школьник.
>> No.143354 Reply
>>143351
Благо, что эту информацию я способен прочесть на обложке. Чем именно этот учебник хорош? В чём его отличия от других?
>> No.143355 Reply
>>143348
> Чем он отличается, например, от того же Фейнмана?
Фейнман хорошо разъясняет физический смысл, но не столь — вычисления.
Другие учебники не смотрел. Кто все эти люди?
>> No.143357 Reply
>>143355
Детлаф - написал с Яворским учебник в 3 томах и справочник по физике для ВТУЗов. Яворского мне советовали выше, как годный курс элементарной физики, но изрядно математизированный.

Л.-Л. - Ландау-Лифшиц. Здесь никаких объяснений не нужно. Сборник - аналог фейнмановских лекций на советском пространстве. Если нужна более подробная информация - в Вики она есть.

Задачник Воробьева-Зубкова - без понятия, кто это. Гуглил задачники, нашел этот, решил спросить здесь.

Ландсберг, как и Яворский, написал курс элементарной физики. Анон выше, который школьник, пользуется и считает годным.
>> No.143361 Reply
>>143357
> Ландау-Лифшиц. Здесь никаких объяснений не нужно.
В самом деле. Только это уже совсем не для школьников.
>> No.143374 Reply
>>143361
Я этого и не утверждал. Лишь желаю уточнить для себя, на чём основана критика данного курса, так как позже хотел бы ознакомиться с ним.
>> No.143378 Reply
>>143374
> на чём основана критика данного курса
Для Л-Л характерны две вещи: концентрированность изложения и движение от общего формализма к частным случаям (как любят делать математики). Кроме того, предполагается, что сопутствующий математический аппарат тебе хорошо известен.
С фейнмановскими лекциями имеет мало общего (лекции практически entry-level).
>> No.143380 Reply
>>143302
Ну это-то все понятно. Если резко поднять его с диска, то оно улетит т.к. инерция, это тоже понятно. Непонятно, почему оно улетит с диска, если он начнет крутиться слишком быстро. Ну вот диск начинает вращение. Со стороны тела на диск будет действовать сила трения тело-диск против движения. По 3 закону Ньютона на диск будет действовать равная по модулю, но противоположная по направлению (значит, направленная по движению) сила трения диск-тело. Что происходит дальше? Ведь должна быть ещё сила трения покоя тела? Или не должна быть? Как находится модуль той силы трения,с которой диск будет действовать на колесо? Кстати, никаких сил натяжения не надо, пусть все тела абсолютно твердые.
>> No.143381 Reply
>>143378
Ошибки в изложении у Л.-Л. есть? Издание какого года наиболее удачное?

И бампану своим предыдущим вопросом, который остался без ответа. "Какие темы из школьного курса геометрии наиболее часто фигурируют в физике? (помимо очевидных: тригонометрии, геометрии треугольника и других плоских/объемных фигур, векторного анализа) Какие учебники позволяют вдоволь проникнуться именно нужными темами? Не слишком математизированные, физики ведь не ставят перед собой цель решать олимпиадные задачи по геометрии."
>> No.143400 Reply
>>143381
> помимо очевидных: тригонометрии, геометрии треугольника и других плоских/объемных фигур, векторного анализа
Да в общем-то, это всё. Геометрия у физиков используется мало, гораздо полезнее учить [линейную] алгебру и анализ/топологию.
>> No.143405 Reply
>>143400
Подборка хороших учебников по этим темам существует?
>> No.143547 Reply
Физик-куны, объясните.
https://yadi.sk/i/7E-KUFV1iAaut
>> No.143552 Reply
Доброаноны, посоветуйте годные видеоролики, в которых ставятся фундаментальные физические эксперименты, например, Опыт Паскаля, Опыт Эрстеда, Опыт Милликена и т.д., а также книги, в которых подробно описан ход эксперимента, его физический смысл и последствия. Очевидно, что многие из опытов нельзя поставить в домашних условиях, поэтому интересует видеоматериал.
>> No.143562 Reply
>>143552
http://www.youtube.com/user/NRNUMEPhI/playlists
Годные опыты, сам разбирался по ним.
>> No.143667 Reply
Кто-нибудь учил механику по Гольдштейну? Какой там мат. аппарат нужен, чтоб пройти от корки до корки? Пока что обнаружил векторный калькулус не заставите меня признать это анализом, физики, но сильно углубиться ещё не успел.
https://docs.google.com/file/d/0B5nvP_eIBydjYWMwMDhhY2QtNzBmOS00NGM5LW[...]/edit
>> No.143727 Reply
Братишки, я понимаю, что платиновый вопрос, но не нашёл доступного моему мозгу объяснения.

И так у нас есть возможность перемещать световой зайчик выше скорости света. Но по какой-то причине он не переносит информацию.

Допустим есть только два-три положения этого зайчика, соответствующие состоянию бита информации.
Очевидно зайчик между положениями может перемещаться больше скорости света, так? Почему нельзя передать информацию? Что будет происходить со «столбом» света, он будет гнуться как пулемётная очередь веером?

Вроде как чую подвох в том, что сами фотоны медленнее скорости света, но состыковать это с замедлением зайчика и передачи на нём информации не могу, потому прошу помощи и наглядного примера без глубокой зауми.

Или всё просто и информацию быстрее скорости света нельзя передать, потому что пачка сигналов, переданных светом долетит до места назначения не быстрее самого света?

Отсюда получается, что не ограничена плотность информации, скорость её считывания и записи, но не передачи. Так?
>> No.143728 Reply
>>143727
Или скорость записи тоже ограничена скоростью близкодействия, то есть можно сделать запись абсолютно любого количества данных разом, но сам отпечаток образуется со скоростью света. Например отпечаток ботинка в грязи — смещение огромного количества молекул, каждой на своё расстояние. Однако смещение происходит не быстрее С. Так?
>> No.143768 Reply
Физик-кун, поясни, пожалуйста, за три физических величины. Это сила тока, напряжение и электрическое сопротивление.

Сила тока, в моём понимании, это количество заряженных частиц (электронов, протонов, дырок), прошедших за 1 с через проводник данного поперечного сечения.

Напряжение - работа, которую нужно приложить электрическому полю, дабы перенести заряд с точки А в точку Б. Можно ли сказать, что к заряду поле "прикладывает силу" и двигает заряд/придаёт ему ускорение? Как во втором законе Ньютона. У меня сложилось впечатление, что напряжение, это "усилие", которое нужно приложить к каждому заряду данной массы, чтобы переместить его.

Электрическое сопротивление - сила тока/напряжение. Получается, что мы рассчитываем "усилие", которое нужно приложить, чтобы данный заряд прошел через данный проводник. И чем больше это усилие, тем больше сопротивление.

Где я обосрался в своих размышлениях?

И ещё один вопрос, на засыпку. Почему, в цепи с последовательным соединением, сила тока в любой части цепи всегда одинакова? Ведь, насколько я понимаю, если есть проводники с разным сопротивлением, необходимо разное время, чтобы заряд одного и того же размера прошел через каждый из них. В таком случае, включение цепи заставит носители заряда двигаться, а потом они затормозят рядом с проводниками и на выходе этих носителей будет меньше? Мои рассуждения, похоже, противоречат закону сохранения электрического заряда, только я не могу осознать, почему?
>> No.143769 Reply
>>143768
> Где я обосрался в своих размышлениях?
Нигде, в принципе. Немного упрощенно, но так оно и есть.
> Почему, в цепи с последовательным соединением, сила тока в любой части цепи всегда одинакова?
Потому что иначе где-то начнут скапливаться лишние электроны, а где-то их не будет. Такая неравномерность неестественна, поэтому сколько электронов вошло, столько и выйдет. По аналогии с трубой, по которой течет вода: где-то эта труба шире, где-то уже, скорость течения там разная, но протекающий там объем (т.е. ток) в единицу времени везде одинаков.
>> No.143770 Reply
File: 1377112377084.jpg
Jpg, 21.82 KB, 480×480 - Click the image to expand
edit Find source with google Find source with iqdb
1377112377084.jpg
>>143769
Спасибо, анон. Ответь тогда на ещё один вопрос. Почему, при параллельном соединении цепи, значение сопротивление приобретает такой вид? Т.е., 1/R+1/R2+... На вики читал, что это связанно с правилами Кирхгофа, но я о них ничего не знаю. Начал заново учить физику, в свои-то года, сейчас дочитываю Пёрышкина, 9 класс. Будет ли в профильных книгах Мякишева за 10-11 классы пояснение данной ситуации?
>> No.143771 Reply
>>143770
Рассматривай это не как сопротивление, а как проводимость. Это величина, обратная сопротивлению. Возвращаясь к аналогии с трубами, проводимость - это площадь сечения трубы. Если у тебя есть две трубы, их проводимость суммируется. То есть как раз 1/R = 1/R1 + 1/R2.
>> No.143773 Reply
>>143770
> На вики читал, что это связанно с правилами Кирхгофа, но я о них ничего не знаю.
Вообще говоря, тебе следует их знать, тем более, что они следуют из простых физических положений.
>> No.143776 Reply
>>143667
> не заставите меня признать это анализом, физики
Отчего ж? Подставь туда многообразия, дифференциальные формы, будет не хуже, чем у математиков.
> Какой там мат. аппарат нужен, чтоб пройти от корки до корки?
Ещё тензоры и самые основы вариационного исчисления.
>> No.143818 Reply
Доброаноны, а за закон Архимеда поясните? Речь идёт о физическом ощущении происходящего.

Мы погружаем тело данной плотности и объёма, к примеру, в воду. Тело имеет вес, а значит, можно рассчитать давление на воду, имея ещё к этому и площадь поверхности соприкосновения с водой. В свою очередь вода, на произвольной глубине, тоже имеет давление, которое рассчитывается по известной формуле. Вектор силы тяжести направлен перпендикулярно вниз. Каждая точка нижней поверхности тела давит на воду, а вода давит на тело. Если давление воды больше, чем давление, производимое весом тела, данное тело всплывает, до тех пор, пока два давления не скомпенсируют друг-друга.

В итоге, когда мы замеряем вес тела с помощью, например, динамометра, мы получаем вес, уменьшенный на это самое давление со стороны воды. Если мы измерим вес в вакууме, а потом в воде, разность показаний и будет силой Архимеда.

Если тело плавает, получается, со стороны воды на каждую точку его нижней поверхности действует давление, равное атмосферному, по закону Паскаля?

И почему в определении говорится, что сила Архимеда равна весу воды, в объёме данного тела? Это понятно, что вес считают по формуле P=pgV. Тело, каждой своей единицей объёма замешает единицу объёма воды в случае, если оно погружено в неё и тонет? Здесь мы рассматриваем ситуацию, когда вода "хочет вернуться на своё место"? Меня именно объём смущает.

Где я обосрался?
>> No.143841 Reply
File: NewCanvas.jpg
Jpg, 64.45 KB, 838×687 - Click the image to expand
edit Find source with google Find source with iqdb
NewCanvas.jpg
>>143818

С одной стороны мы имеем вес тела, с другой вес выдавленной им воды. Как на весах.
Если тело плавает, значит всё оно весит столько, сколько вытеснила воды его погружённая часть.
Если тело погружено полностью, то его объём равен объёму вытесненной воды.

Там, где в воду погружено тело, воды нет, можно считать закрытой трубкой, получается пикрилейтед, эдакие гидравлические весы.

Вода давит на тело, чем глубже, тем сильнее, но разница давлений при полном погружении тела всегда одна, по высоте тела. Градиент давления пытается выдавить тело вверх против силы тяжести.
По сути борются тяжесть воды и тяжесть тела.

Вода стремится не занять прежнее место а растечься, заняв состояние с минимальной потенциальной энергией, то есть устойчивое. Вот и давит на всё подряд.
>> No.143980 Reply
File: 1241281415925.jpg
Jpg, 60.98 KB, 426×421 - Click the image to expand
edit Find source with google Find source with iqdb
1241281415925.jpg
Аноны, помогите. Я попал в сложную ситуацию и мне не у кого больше спросить.
Знаю, вы не будете смеяться, поэтому пришел сюда.
Расскажи мне, анон, как же меняются времена года на нашей планете?
Википедия говорит, что всему виной наклон земной оси. Но ведь шар приблизительно круглый. Его ось как не наклоняй, а суммарное тепло, полученное планетой, будет одинаковое.
Тут я подумал, что, возможно, некоторые мои представления о мире ошибочны. А вдруг тепло получается одно, но оно распределяется по планете неравномерно. Это значит, что у нас сейчас лето, а в южном полушарии зима. В таком случае июнь, июль и август у них холодные месяцы и везде лежит снег. Неужели это так?
Блин. Загуглил. Оказывается реально так. Там сейчас зима оказывается.

Не знаю, почему, но я всегда думал, что времена года одинаковые на Земле и связано это с тем, что планета просто в некоторые моменты времени дальше от солнца, чем в другие. Она как бы летает по овалу и Солнце находится не совсем в центре.

Ужас. Может это просто сон? Или я попал в другой мир? Но до сегодняшнего дня я ни разу не встречал опровержение своей теории.
>> No.143986 Reply
>>143980
> Но ведь шар приблизительно круглый
   Погугли геоид.
> Не знаю, почему, но я всегда думал, что времена года одинаковые на Земле и связано это с тем, что планета просто в некоторые моменты времени дальше от солнца, чем в другие.
Распространённое заблуждение, которое есть у тех, кто не прочёл ни одной книжки по астрономии.
> Но до сегодняшнего дня я ни разу не встречал опровержение своей теории.
Больше книжек по физике читай - узнаешь ещё много нового. Ну или школу не нужно (было) прогуливать.
>> No.143994 Reply
>>143980
Лол. Читай хотя бы Жюль Верна.
>> No.143995 Reply
>>143980
> Она как бы летает по овалу и Солнце находится не совсем в центре.
Эксцентриситет земной орбиты равен где-то 0,01671123. А теперь давай попросим вольфрамальфу нарисовать соответствующий по форме эллипс:
http://www.wolframalpha.com/input/?i=polar+plot+r%3D1%2F%281+%2B+0.016[...]+2+pi
То, насколько отличаются апогелий и перигелий относительно длины большой полуоси ты можешь посчитать и сам.
>> No.144020 Reply
File: 1287350630391.jpg
Jpg, 1589.09 KB, 2000×1406 - Click the image to expand
edit Find source with google Find source with iqdb
1287350630391.jpg
Доброаноны, пришел к вам с несколькими вопросами. Всё по порядку.

Длина волны, согласно определению, равна расстоянию, которое проходит фронт волны за промежуток времени, равный периоду колебания. Если мы будем рассматривать фронт волны как колеблющийся маятник, можно ли утверждать, что длина волны равна расстоянию в 4 амплитуды колебания данного маятника?

"Тригонометрия" Гельфанда будет хорошим учебником по тригонометрии? Мне хватит его на университетский курс физики или необходимы ещё учебники по данной теме?

Какие книги можно почитать, чтобы точней вникнуть в суть понятия "энергия" во всём её разнообразии? Не околофилософского толка, а именно физического.

Буду премного благодарен за ответы.
>> No.144023 Reply
>>143980
Никуда не уходи. Вернусь с работы и если подробного ответа тут не будет, запилю сам, с картинками(если не сильно там задержусь).
>> No.144035 Reply
>>144020
> длина волны равна расстоянию в 4 амплитуды колебания данного маятника
Нет, амплитуда от частоты/длины волны никак не зависит. Длина волны зависит лишь от скорости (c для света в вакууме) и частоты, всё.
> "Тригонометрия" Гельфанда будет хорошим учебником по тригонометрии? Мне хватит его на университетский курс физики или необходимы ещё учебники по данной теме?
Более чем. Фактически, там намного больше, чем тебе понадобится.
> Какие книги можно почитать, чтобы точней вникнуть в суть понятия "энергия" во всём её разнообразии? Не околофилософского толка, а именно физического.
Фейнмановские лекции по физике же.
>> No.144050 Reply
Какой задачник пользовать совместно с Сивухиным?
>> No.144058 Reply
>>144050
Овчинкин.
>> No.144060 Reply
>>144020
Вот это каша на пикче.
>> No.144177 Reply
>>137425
Доброго времени суток физик-анону. Тут намедни посетила забавная теория разнеси её в пух и прах интересно почитать будет. Теория : есть мнение что структурированные частицы (атомы а потом и молекулы) состоя из более мелких частиц обладают свойством в каком то виде сохранять информацию о том в каких структурах они уже существовали и соответственно при распаде структуры в которой они были до этого они будут стремиться(это из электричества) по пути наименьшего сопротивления создавая при этом структуры условия в которых будут максимально схожи с теми в которых они были до этого. С меня как обычно.
>> No.144179 Reply
>>144177
> разнеси её в пух и прах
У тебя нет абсолютно никакой модели чтобы делать какие-либо конкретные предположения. Т.е в твоих словах физического смысла нет.
>> No.144181 Reply
File: dg-vsMzuyco.jpg
Jpg, 62.59 KB, 431×600 - Click the image to expand
edit Find source with google Find source with iqdb
dg-vsMzuyco.jpg
>>137425
Продублирую в этот тред.
Привет, универчач. Посоветуй учебник по физике, чтобы подтянуть общешкольный курс перед вузом. Специальность с физикой не связана, но в некотором количестве присутствует в программе. Физику знаю довольно хуево, хочу подтянуть.
>> No.144209 Reply
>>144181
Уже с каждой стороны облобзали каждый достойный учебник. Буквально в каждом треде один и тот же вопрос обсуждают 3-5 раз. Сделайте ньюфагам faq что ли.
>> No.144247 Reply
>>144023
Ты очень долго работаешь..
мимо-другой-кун
>> No.144331 Reply
Аноны, задач с Ландсберга мне хватит для элементарного курса? Или нужен ещё задачник? Если да, какой?
>> No.144365 Reply
>>144331
Быть может Рымкевич 10-11?
>> No.144375 Reply
File: WtWRI7iGB9Y.jpg
Jpg, 58.17 KB, 600×848 - Click the image to expand
edit Find source with google Find source with iqdb
WtWRI7iGB9Y.jpg
Почему проводники нагреваются при прохождении по ним тока? Я всегда думал, что это трение, но недавно мне сказали, что это не так и сказали, что это из-за фотоэффекта. Но ведь фотоэффект это когда электроны вылетают из тела под действием света, причем он тут?
Как с точки зрения физики объясняется жизнь? Ведь все мы - кучка хаотично электронов, вращающихся вокруг комков из протонов и нейтронов. Почему вообще существуют какие-либо структуры? Ведь электроны, атомы и прочие штуковины движутся беспорядочно, разве мир не должен в таком случае представлять из себя безжизненный хаос радномно движущихся частиц?
Почему электроны не падают на ядра атомов? Иди хотя бы не сталкиваются с ними во время движения?
Почему меняются времена года?
>> No.144376 Reply
>>144375
> Ведь все мы - кучка хаотично электронов, вращающихся вокруг комков из протонов и нейтронов.
> Ведь электроны, атомы и прочие штуковины движутся беспорядочно
Нихуя.
>> No.144379 Reply
>>144375
> Почему электроны не падают на ядра атомов?
Электрон на самом деле не похож на планету, летающую вокруг звезды. Электрон похож на каплю жидкости, в которой плавает ядро. Форму этой капли ты видел на уроках химии - электронные облака, соответствующие p-орбиталям, похожи на гантели, например.
>> No.144381 Reply
>>144379
> Электрон похож на каплю жидкости, в которой плавает ядро.
Нихуя. Зачем эти туманные аналогии? Пусть учит квантовую механику, если хочет знать о таких вещах.
>> No.144399 Reply
File: 417863_html_100010e1.png
Png, 110.24 KB, 389×449 - Click the image to expand
edit Find source with google Find source with iqdb
417863_html_100010e1.png
>>144381
> Нихуя. Зачем эти туманные аналогии?
Может потому что никто не знает, как выглядит электрон и что это такое?
Познание человеку доступно только через модели (аналогии).

Пока что известная аналогия довольно сложная. Есть часть пространства вокруг ядра, в области которого существует вероятность обнаружить электрон при попытке наблюдения (взаимодействия). На картинке каждая точка — место наиболее вероятного обнаружения в отдельный момент времени. То есть в облаке есть распределение вероятностей по объёму. А электрон «размазан» во времени и пространстве, и «выглядит» как облако. Тащемта в мире всё такое: оно как бы есть и его как бы нет.
>> No.144401 Reply
>>144375
> Почему электроны не падают на ядра атомов?
Потому что обладают кинетической энергией.
> Иди хотя бы не сталкиваются с ними во время движения?
Размеры электрона пренебрежимо малы по сравнению с размерами ядер.
>> No.144431 Reply
File: Breaking-News_cover-2.jpg
Jpg, 135.53 KB, 400×600 - Click the image to expand
edit Find source with google Find source with iqdb
Breaking-News_cover-2.jpg
>>137425
> удалось смоделировать процесс с минус трением с поглощением тепла
>> No.144474 Reply
File: zadacha.JPG
Jpg, 29.21 KB, 598×113 - Click the image to expand
edit Find source with google Find source with iqdb
zadacha.JPG
Пикрилэйтед. Не понимат как решит. Причём тут алюминий и площадь сечения? Как расположена горизонтальная линия? Считается, что она держится на чём-то (в тисках или просто лежит на поверхности), или падает вниз? Думаю, что тут нужны закон Ампера и вес по объёму (из площади сечения на метр) и плотности.
>> No.144477 Reply
File: [HorribleSubs]-Haiyore-Nyaruko-san-05-[1080p].mkv_.jpg
Jpg, 164.39 KB, 1920×1080 - Click the image to expand
edit Find source with google Find source with iqdb
[HorribleSubs]-Haiyore-Nyaruko-san-05-[1080p].mkv_.jpg
>>144474
Его надо положить горизонтально перпендикулярно к другому проводнику. Потому что если не перпендикулярно, то он будет двигаться под действием магнитного поля, если не горизонтально, то гравитации.
>> No.144506 Reply
>>144477
Как ты себе это представляешь, "горизонтально перпендикулярно к другому проводнику", если этот второй проводник уже расположен горизонтально, согласно условию задачи?
>> No.144513 Reply
>>144506
Горизонтально - это плоскость, омич. В ней к прямой как раз можно провести ровно один перпендикуляр.
>> No.144516 Reply
>>144477
Но ведь оно и так будет двигаться под действием магнитного поля из-за наличия площади. Располагаемый проводник имеет площадь же, а не уже горизонтальный.
>> No.144517 Reply
>>144516
Не будет. Силовые линии магнитных полей двух проводников все равно будут везде перпендикулярны, то есть взаимодействия не будет.
>> No.144535 Reply
Антоши, подскажите, какой перевод "Математических начал натурфилософии" Ньютона на русский язык наиболее приемлем? Изучение латыни присутствует, как одна из целей, но пока я не могу в неё, хотелось бы ознакомиться с сим замечательным трудом.
>> No.144574 Reply
>>144535
Академический, изданный АН СССР в серии "классики".
>> No.144708 Reply
Аноны, мне нужно быстро повторить школьную физику, точнее механику. На моём мехмате внезапно начался термех, а я не то что задачки решать, формулы все забыл.
Какую литературу посоветуете?
>> No.144709 Reply
>>144708
А как теормех связан со школьной механикой? Тут Ландавшиц нужен.
>> No.144739 Reply
>>144709
Гантмахера читай. И термех никак не связан со школьной физикой.
>> No.144821 Reply
File: 14407702437842.jpg
Jpg, 85.30 KB, 640×496 - Click the image to expand
edit Find source with google Find source with iqdb
14407702437842.jpg
Читаю про излучение/испарение Хокинга и не понимаю, если на горизонте событий образуется пара частица - античастица, то только один объект покидает поле дыры, а второй по идее должен увеличивать её массу, не? Какое тогда это испарение?
Что случится если подвергнуть черную дыру бомбардировке антиматерией? Неужели её масса станет меньше и если это происходит только за счет античастиц, то почему об этом нигде не пишут?
>> No.144824 Reply
>>144708
> На моём мехмате
Какой курс, какая группа?
мимо-с-мехмата
>> No.144825 Reply
>>144821
Частицы создаются за счёт энергии дыры. Вот, предположим, такая схема.

Арбуз стоит сто рублей. У тебя есть миллион рублей.
Ты покупаешь два арбуза. Один арбуз ты продаёшь (без прибыли), а другой внезапно куда-то улетает.
И теперь у тебя девятьсот девяносто девять тысяч девятьсот рублей.
Ты снова покупаешь два арбуза. Один продаёшь, а другой улетает.
У тебя девятьсот девяносто девять тысяч восемьсот рублей.
И так до тех пор, пока у тебя не останется сто рублей и всё не закончится.
>> No.144914 Reply
>>144825
Какая глупость.Фи. Вас бы не существовало как и всей материи с такими трактовками в этом мирке.
>> No.144920 Reply
File: stephen-hawking.jpg
Jpg, 47.00 KB, 468×382 - Click the image to expand
edit Find source with google Find source with iqdb
stephen-hawking.jpg
>>144825

Это вэликолепно! Просто замэчательно. Я тэпэрь всэм это буду так объяснять, дарагой.
Только арбузы у тэбя какиэ-то дэшовые.
>> No.144972 Reply
File: zad.JPG
Jpg, 31.19 KB, 616×128 - Click the image to expand
edit Find source with google Find source with iqdb
zad.JPG
Это снова я на связи. Задачка пикрилэйтед.
Я думаю, что тут надо применить закон Фарадея в таких формах:
rot E = -dB/dt
ЭДС = S(Edl) = -dФ/dt
dl = ЭДС/E
А что дальше делать? Я бака, да.
>> No.145072 Reply
File: 13525477717663.jpg
Jpg, 37.78 KB, 619×619 - Click the image to expand
edit Find source with google Find source with iqdb
13525477717663.jpg
Аноны, я не знаю вариационного исчисления. А без него трудно читать теормех. Подкиньте методичку или статью про вариации. Много не надо, только чтобы в теормехе легче разбираться было. В матане более менее шарю. Читаю Ландау.
>> No.145092 Reply
>>145072
У Гельфанда довольно простая книжка по вариационному исчислению.
>> No.145099 Reply
File: 1253305190278.jpg
Jpg, 63.03 KB, 500×373 - Click the image to expand
edit Find source with google Find source with iqdb
1253305190278.jpg
>>145092
Спасибо! Идеально просто.
>> No.145105 Reply
>>145072
> термех
> Ландау
Ящитаю, что это далеко не лучший выбор. Далеко.
>> No.145107 Reply
>>145072
А что такое вариационное исчисление?
мимо-олимпиадник по теормеху
>> No.145108 Reply
File: i-dont-want-to-live-on-this-planet-anymore-11372-1.jpg
Jpg, 386.04 KB, 1920×1200 - Click the image to expand
edit Find source with google Find source with iqdb
i-dont-want-to-live-on-this-planet-anymore-11372-1.jpg
>> No.145123 Reply
>>145105
Или поясняй или ты не голоморфный.
>> No.145335 Reply
File: 6789d1898d329a748f349f6f5509ae36.jpg
Jpg, 112.57 KB, 900×1280 - Click the image to expand
edit Find source with google Find source with iqdb
6789d1898d329a748f349f6f5509ae36.jpg
Есть твердотельщики или сочувствующие? Объясните нубу как интегрировать по зоне Брилюэна, в самом общем и нескольких более высокосимметричных случаях.
Спасибо.
>> No.145463 Reply
Антоши, что скажете относительно курсов теоретической физики Планка и Зоммерфельда? Они безнадежно устарели или все ещё актуальны? Есть ли у них преимущества перед Л.-Л.?

"Высшая математика" Зельдовича даст возможность почерпнуть необходимые знания в области дифференциального и интегрального исчисления или необходимы какие-то более фундаментальные труды?

Те требования по математике, которые указаны в теорминимуме Ландау выглядят не сильно впечатляющими. Если попотеть над представленными темами, можно ли будет адекватно воспринимать выкладки по теорфизике?
>> No.145736 Reply
Здраствуйте, доброняши. Дайте, пожалуйста, литературы по общей физике и механике на первые пару курсов вуза.
>>145735 -кун
>> No.145742 Reply
>>145736
Какой-нибудь учебник по общей физике (Савельев, например), Л.-Л, «Математические методы классической механики» Арнольда.
>> No.145984 Reply
Анон, помогай, у меня сейчас голова сломается.
"Математический маятник укреплен на тележке. Тележка без трения скатывается по наклонной плоскости с углом a=30°. Найти угол отклонения нити от нормали к плоскости".
В общем, тележка очевидно скатывается с ускорением равным g*sin(a). Значит, относительно тележки на шарик действуют силы тяжести, натяжения нити и инерции. Проведем ось Ох параллельно плоскости и спроецируем туда 2 закон Ньютона.
Получается, что:
Tsin(b)+mgsin(a)=mg*sin(a) сумма проекций натяжения нити и силы тяжести равна массе на ускорение
И тогда угол отклонения b равен нулю. Где ошибка?
>> No.146109 Reply
File: 14402709002380.jpg
Jpg, 145.43 KB, 1920×1200 - Click the image to expand
edit Find source with google Find source with iqdb
14402709002380.jpg
Здравствуй, доброчан! Сегодня пришел с физики, на следующей паре у меня контрольная. Дело в том, что я вообще не могу в решение задач по физике, я могу записать дано, могу иногда описать картину происходящего, но как правило до конца довести задачу я не могу. Занимаемся мы по задачнику Иродова, кому интересно могу скинуть последнюю задачу что я не понял. Так вот, есть ли какой-то более легкий задачник по физике, чтобы быстро въехать в решение задач? Ну или чего еще можно придумать? Готов заниматься почти 24\7.
>> No.146111 Reply
>>146109
Забыл добавить, попробую сейчас прочитать Савельева, но задачник по прежнему интересует.
>> No.146517 Reply
Пришел сюда я с реквестом книг/статей научных на две темы.
Первая - ответ на вопрос "природа электростатических сил"
Вторая - некое введение в квантовую физику, физику элементарных частиц и физику конденсированых сред.
И да, крайне желательно, к книгам содержащим матан приложить книги по этому самому матану.
Заранее благодарю.
>> No.146647 Reply
Так, анон. Генераторы переменного тока, турбины на ТЭЦ, ЭДС индукционного тока и тому подобные штуки. Как это работает? Вот генератор, в нем пар крутит турбины, на которые приклеен магнит, и все это находится в проводящем контуре? И возникает ЭДС индукции. Почему? Почему изменение потока заставляет электрончики бегать? И где именно в контуре создается эта разность потенциалов? На каких двух точках? Почему электроны внезапно начинают выбегать из контура в мир вместо того, что бы просто перейти в точку с меньшим потенциалом по кратчайшему пути? Почему они не кончаются в контуре?
>> No.147491 Reply
File: tumblr_nxxhwrQDFT1qa5uhxo1_540.jpg
Jpg, 106.18 KB, 540×625 - Click the image to expand
edit Find source with google Find source with iqdb
tumblr_nxxhwrQDFT1qa5uhxo1_540.jpg
Аноны, такая задача.
"Вдоль стенки цилиндрической трубы идет постоянный ток силы I. какова напряженность магнитного поля H внутри и вне трубы."
Я понимаю как рассчитать B. Но причем тут H? Напряженность же вроде в веществе. Как H и В будут соотноситься тогда?
>> No.147498 Reply
>>147491
> Напряженность же вроде в веществе.
Можно и без вещества. В СИ H = B/μ₀, в СГС H = B.
>> No.147499 Reply
>>147498
Ок, то есть в задаче можно просто найти B и все? Это ответ будет?
>> No.147678 Reply
Насчёт электродинамики советую http://lectoriy.mipt.ru/course/Physics-Electricity-14S#lectures
Удобнее всего.
>> No.147687 Reply
>>147499
Найти B и поделить на μ₀ (у тебя же задача в единицах СИ?).
>> No.147916 Reply
File: 1.jpg
Jpg, 26.21 KB, 375×500 - Click the image to expand
edit Find source with google Find source with iqdb
1.jpg
Посоветуйте, пожалуйста, годную книгу по теормеханике, доходчивую, которую можно было бы читать без арнольдовского тривиума и PhD по математике.
>> No.147935 Reply
>>146647
Индукционный ток, вообще говоря, не является потенциальным.
>> No.148356 Reply
>>147916
Это ты намекаешь, что ЛЛ не подходит? Я боюсь, что то, что там изложено, и является определением теормеханики. То есть без лагранжианов, вариационного исчисления и подобного ты просто никуда не уедешь.
>> No.148357 Reply
>>147916
Л-Л, например.
>> No.148368 Reply
Как стать учёным-физиком?
>> No.148370 Reply
File: t1-14059b84-0c6a-...
Jpg, 121.01 KB, 610×720
edit Find source with google Find source with iqdb
t1-14059b84-0c6a-4c58-82b2-83ecc31452ea.jpg
File: tumblr_ny1op4VKiU...
Png, 1191.11 KB, 818×728
edit Find source with google Find source with iqdb
tumblr_ny1op4VKiU1ut1mr9o1_1280.png

Анон, ты когда нибудь задумывался, почему нейтронные звезды существуют, ведь период полураспада свободного нейтрона 611 секунд.
   Наверняка ведь кто-то умный уже выяснил почему, но до меня такая новость еще не дошла.
>> No.148371 Reply
>>148370
Потому что нейтроны в нейтронной звезде не являются свободными. Так же как и в атомных ядрах.
>> No.148372 Reply
>>148371
То есть гипотетическое "ядро" из двух нейтронов должно быть более стабильным чем 2 нейтрона по отдельности?
>> No.148374 Reply
>>148372
Ноуп, "ядро" из двух нейтронов нестабильно потому что у него есть более выгодный энергетический уровень: протон и нейтрон, и оно на этот уровень с испусканием электрона немедленно перейдет. Распаду должен мешать некий потенциальный барьер, делающий распад энергетически невыгодным. Например протон по-соседству. Если бы нейтрон распадался рядом с протоном, то оставалось бы ядро из двух протонов, которое должно распидорасить, а энергия, с которой его распидорашивает, не может взяться из ниоткуда. Поэтому рядом с протонами нейтроны не распадаются. Кроме бета-минус распада тяжелых ядер, когда протонов на всех не хватает. В случае нейтронной звезды роль этого потенциального барьера играет гравитация. Энергия вылетевшего электрона оказывается слишком мала для того чтобы покинуть звезду, поэтому он не вылетает. Или вылетает и возвращается. Если он это сделал за планковское время, разницы вообще нет.
Впрочем, это мое дилетантское мнение, и я не исключаю, что сейчас сюда придет физик-кун и выльет на меня ведро помоев. Хотя, физик-кунов в этом треде хронически недостает.
>> No.148375 Reply
>>148374
Ах да, не стоит еще забывать что в нейтронных звездах еще и куча протонов. Иначе бы у них не было таких магнитных полей. Я вообще к ним отношусь как к огромным атомным ядрам.
>> No.148378 Reply
>>148370
Ну вот смотри, в нейтронной звезде внешний слой состоит преимущественно из нейтронов, позитронов и электронов. Какие-то нейтроны распадаются, а какие-то электроны с позитронами сталкиваются и объединяются в нейтроны. Эти два процесса и уравновешивают друг друга. В более глубоких слоях добавляются ещё мюоны, гипероны и совсем уж неведомая фигня.
>> No.148379 Reply
>>148378
> какие-то электроны с позитронами сталкиваются и объединяются в нейтроны
Ты явно что-то путаешь. И очень сильно путаешь.
>> No.148380 Reply
>>148374
> Если бы нейтрон распадался рядом с протоном, то оставалось бы ядро из двух протонов, которое должно распидорасить, а энергия, с которой его распидорашивает, не может взяться из ниоткуда
Ну, примерно так. У протона с нейтроном в ядре есть отрицательная энергия взаимодействия (они притягиваются, обмениваясь мезонами). Если нейтрон распадётся, то энергия системы, действительно, увеличится, потому что два протона стабильную конфигурацию. Поскольку система в целом стремится к состоянию с наименьшей энергией, они и не распадаются.
>> No.148381 Reply
>>148379
Вместо позитронов везде должны быть протоны, да. Оговорился.
>> No.148384 Reply
>>148374
>>148380
Спасибо, друзья. Я так примерно и думал, но нет ли ничего такого почитать по этой теме, что может понять (хотя бы на уровне идеи) человек, имеющий физическое образование на уровне 11го класса?
>>148375
Про это не знал, спасибо.
>> No.148386 Reply
File: няша1312.jpeg
Jpeg, 258.18 KB, 1000×662 - Click the image to expand
edit Find source with google Find source with iqdb
няша1312.jpeg
>>148384
Вот вам няшу за участие
>> No.148420 Reply
А я как всегда к вам с вопросом.
Гравитационное линзирование. А точнее этот самый крест Айнштайна. Почему крест? Почему не кольцо? Можно подумать, что свет распространяется только вдоль 4х линийкоторые и образуют крест, а не во всех направлениях.
>> No.148428 Reply
File: 13209933052938.jpg
Jpg, 968.73 KB, 1947×2278 - Click the image to expand
edit Find source with google Find source with iqdb
13209933052938.jpg
>>148357
Л-Л? Не знаю, мне кажется, вы шутить изволите. Надо же, наконец какое-то движение спустя чуть не месяц, лол
>> No.148429 Reply
File: 13535860968512.jpg
Jpg, 136.88 KB, 800×600 - Click the image to expand
edit Find source with google Find source with iqdb
13535860968512.jpg
>>148356
Да, лагранжианы, гамильтонианы как раз и есть то, что надо. Я имел в виду что-то не чересчур математизированное, как у того же Арнольда. Просто у ЛЛ тоже довольно своеобразная слава как будто. Вроде как не самое доходчивое чтиво.
>> No.148443 Reply
>>148428
>>148429
ЛЛ тяжеловат, но читаем. Не думаю, что ты найдёшь что-то лучше с теми же достоинствами.
>> No.148445 Reply
>>148443
Тогда спасибо, попробую ЛЛ.
>> No.148568 Reply
>>148420
Не знаю, может быть здесь замешаны какие-нибудь хитрые каустики?
Я слышал, свет любит по ним распространяться.
>> No.148773 Reply
File: 14493555886151.jpg
Jpg, 274.07 KB, 1280×836 - Click the image to expand
edit Find source with google Find source with iqdb
14493555886151.jpg
Анон, подскажи пожалуйста хорошую книжку по физике/астрономии/квантовой физике, затрагивающую нерешенные вопросы по сей день будоражащие ученые умы? Желательно с самым широким охватом тем, без требований к глубоким знаниям, не сильно упрощенную, и написанную с увлечением.

Краткая история времени доставила. Лекции Фейнмана доставили.

Охота чтобы затрагивалось подобное:

ruwiki://Ложный_вакуум

ruwiki://Гамма-всплеск

ruwiki://Многомировая_интерпретация
>> No.148966 Reply
Расскажите, кто сведущ в физике. Вот есть абсолютный ноль -273 С, а есть ли абсолютный максимум? Как далеко может продолжаться нагревание материи? И почему такая большая разница между минимумом и максимумом, если заметить, что некоторые звёзды имеют температуру +20000 С?
>> No.148967 Reply
>>148966
Нет максимума. Твоя ошибка заключается в том, что ты оперируешь понятием "на сколько". Это не важно. Важно во сколько раз. Поверхность звезды не на 20кС теплее поверхности земли, а в 100 раз. И предела нет, хотя при достаточно высоких температурах, безусловно, случится что-то интересное, например, кварк-глюонная плазма.

>>148773
> Ложный вакуум
Интересная концепция, но не думаю, что она затрагивается в учебниках. Попробуй почитать статьи, в которых вводилось это понятие.
> Гамма-всплеск
Непонятное явление. Если хочешь почитать про столкновения нейтронных звезд, ищи статьи, в которых авторы изучают (моделируют/etc) столкновения нейтронных звезд, и предлагают такие явления на роль кандидатов в источники гамма-всплеска.
> Многомировая интерпретация
Заткнись и считай. Все интерпретации квантовой физики неверны. Верна только математика в ее основе.
>> No.148968 Reply
>>148966
Планковская температура.
>> No.148976 Reply
>>148966
> И почему такая большая разница между минимумом и максимумом, если заметить, что некоторые звёзды имеют температуру +20000 С?
А потому, что ты основываешься на шкале температур в градусах Цельсия, где за 0 принят не абсолютный ноль как у Кельвина, а просто температура плавления водяного льда. С таким же успехом можно было взять за ноль температуру плавления азота и получить еще более меньшую разницу между абсолютным нулем и соответствующей температурой плавления. Проще говоря, нету никакой отрицательной температуры, есть абсолютный 0, когда атомы условного вещества прекращают колебаться, а также есть отличная от 0 положительная температура, когда уже возможно тепловое движение.
>> No.149002 Reply
>>148966
темпрература - это скорость молекул. абсолютный ноль = состояние покоя.

хотел сказать "нету максимума" но задумался про скорость света. является ли максимальной температура соответствующая движению молекул со скоростью света, или на том уровне это понятие не применимо?
>> No.149003 Reply
>>148967
> Попробуй почитать статьи
я эти понятия как пример процитировал. хочется похожего и доступным текстом, но непонятно с какой стороны подходить. олсо нуб.
>> No.149005 Reply
>>149002
Температура показывает не скорость, а энергию частиц. Как релятивистская скорость связана с энергией, ты можешь почитать в википедии в статье про СТО.
В принципе, ограничение на энергию отдельной частицы есть - если ее релятивистская масса достигает планковской, она по идее должна превратиться в черную дыру (не то чтобы это было возможно хоть как-то экспериментально проверить, но такой смысл закладывается в понятие планковской массы). Температура, при которой средняя энергия частиц такова, называется планковской, и в принципе представляется собой somehow предел, хотя если применять термин "температура" к оставшемуся от такого нагрева скоплению черных дыр, не вижу причин чтобы она не росла дальше.
>> No.149006 Reply
>>149003
Если тебя интересуют именно научно-популярные вещи, смело бери любую книгу Митио Каку, его интересно читать, хотя надо немного фильтровать. Ну и конечно же Хокинг - наше всё.
>> No.149014 Reply
Между прочим, минимальной температуры тоже нет. Абсолютный ноль - только предел. Но он недостижим за конечное количество шагов.
>> No.149375 Reply
File: 300px-Acceleration-due-to-Gravity-on-Earth.png
Png, 32.58 KB, 300×354 - Click the image to expand
edit Find source with google Find source with iqdb
300px-Acceleration-due-to-Gravity-on-Earth.png
>>137425
Задавал свой вопрос в треде глупых вопросов, но там мне ничего толком не ответили. Попробую тут.
Аноны, вот смотрите. Земля крутится вокруг своей оси. Если мы находимся в системе отсчета, связанной с Землей (т.е. неинерциальной), то на тело, падающее с некоторой высоты, действует сила Кариолиса и центробежная сила (в следствие последней g эффективное направлено не в центр Земли, а чуть ниже (если мы в северном полушарии)). Это объясняет небольшое смещение траектории тела в сторону экватора при падении. Тут все понятно. А если мы перейдем в систему отсчета, в которой Земля вращается? Т.е. в инерциальную. Какие силы тут объясняют смещение в сторону экватора? Ведь тут уже нет центробежной силы, из-за которой g эффективное направлено не в центр. А какие силы тогда заставляют тело падать именно так?

P.S. Интересует именно смещение в сторону экватора. Про смещение на восток (в северном полушарии) я понимаю, что просто Земля прокручивается.
>> No.149378 Reply
File: _Безымянный.png
Png, 4.10 KB, 427×340 - Click the image to expand
edit Find source with google Find source with iqdb
_Безымянный.png
>>149375
Никакие. Дело в том, что у тело тоже движется вместе с вращением замли, то есть обладает скоростью. Таким образом, когда его отпускают, оно без гравитации должно было бы продолжать двигаться по прямой, а Земля-то крулая, и поэтому в плоскости вращения земля из-под него как бы уходит и оно проделывает путь, больший чем если бы земля была плоской (пикрилейтед).
Вообще, следует понимать главное: Не бывает никакой центробежной силы. И силы Кориолиса не бывает. Это абстракции, описывающие инерцию в неинерциальной системе отсчета.
>> No.149380 Reply
>>149378
Хм. Спасибо большое, я подумаю над этим. Но вроде все понятно теперь, да.
Да, я понимаю, что это абстракции, поэтому и интересно, почему так происходит, если обойтись без них.
>> No.149974 Reply
File: 1233753031854.jpg
Jpg, 20.50 KB, 493×424 - Click the image to expand
edit Find source with google Find source with iqdb
1233753031854.jpg
А вот смотрите, какая ерунда, господа физики. Вот есть реликтовое излучение и как будто говорят, что это заметно "покрасневшие" фотоны из далёкого прошлого. Но если для фотонов E ~ m, то значит, "краснея", фотон теряет массу/энергию. А куда эта энергия девается-то? Разве это не противоречит закону сохранения энергии вообще?
>> No.150006 Reply
>>149974
Это значит, что источник фотона движется в направлении от приемника.
>> No.150224 Reply
Как вы так укатились?! Дальше секса-с-тян треда. Дальше математики. Да даже дальше трапо-треда. воу.
Аноны, есть один кун 11 класса, и этот кун - я. И мне нужна помощь с сами понимайте чем с физикой. А именно просто вопросы по некоторым задачам, которые я не могу решить. Я не так, что бы тупой, но проблемы бывают. А спросить не у кого совсем. Так что если вдруг кто-то захочет потратить время и объяснить мне какое-то кол-во задач может даже и не маленькое буду рад тут :
helpless.guy.need.physics2@mail.ru
>> No.150225 Reply
>>150224
Почему бы просто не спросить здесь? Пости задачи сюда, пиши свои мысли по их решению, анон намекнет, куда думать дальше.
>> No.150255 Reply
File: 1.jpg
Jpg, 757.53 KB, 1552×2592
edit Find source with google Find source with iqdb
1.jpg
File: 2.jpg
Jpg, 632.45 KB, 2592×1552
edit Find source with google Find source with iqdb
2.jpg
File: 3.jpg
Jpg, 731.61 KB, 2592×1552
edit Find source with google Find source with iqdb
3.jpg
File: 4.jpg
Jpg, 790.25 KB, 2592×1552
edit Find source with google Find source with iqdb
4.jpg
File: 7.jpg
Jpg, 643.46 KB, 2592×1552
edit Find source with google Find source with iqdb
7.jpg

>>150225
Ибо вопросов чуть больше, чем очень много. Вот например, только две задачи, а уже все 7 слотов для файлов заняты. 11.1 и 11.2 . Условия тоже прилагаю. Ответ почему-то просто отличается от данного в учебнике. В чем проблема\ошибка я не знаю. А хочу знать.
>> No.150257 Reply
>>150255
В обоих задачах у тебя все правильно, это в ответах ересь. Сам подумай, откуда у них взялась двойка во второй задаче? Единственное место, откуда ее можно было получить - из корня коэффициента. Они и получили. А 4 забыли убрать. А ты не забыл, молодец, 5. Давай задачи посложнее, эти неинтересные.
>> No.150307 Reply
File: 3.jpg
Jpg, 718.47 KB, 1552×2592
edit Find source with google Find source with iqdb
3.jpg
File: 4.jpg
Jpg, 729.97 KB, 1552×2592
edit Find source with google Find source with iqdb
4.jpg
File: 1.jpg
Jpg, 813.06 KB, 2592×1552
edit Find source with google Find source with iqdb
1.jpg
File: 5.jpg
Jpg, 733.93 KB, 1552×2592
edit Find source with google Find source with iqdb
5.jpg
File: 2.jpg
Jpg, 887.32 KB, 1552×2592
edit Find source with google Find source with iqdb
2.jpg

>>150257
Ну ды. Например, 11.5 и 11.12. В 11.5 я могу разве что найти ускорение левого шара по вертикали, и оно совпадает с ответом, но дальше что-то не оч. У первого шара пропадет сила натяжения нити, но за счет чего начнет двигаться правый шар? Ведь ускорение надо найти непосредственно после пережигания, значит стержень между ними ещё не успел растянутся или ещё что-то.. А 11.2 я собственно решил, но ответ в учебнике странный будем считать, что его нет, так что я не уверен. И 11.11. Потенциальная энергия и её изменения считаются относительно 1 любого шара, или относительно всех сразу?
>> No.150308 Reply
File: hint.png
Png, 8.06 KB, 463×404 - Click the image to expand
edit Find source with google Find source with iqdb
hint.png
>>150307
> стержень между ними ещё не успел растянутся
Он же нерастяжимый, бака.
Фишка в том, что хотя в момент пережигания уравнения и приобретают ненулевую правую часть, точка B начинает двигаться по окружности, следовательно ее ускорение будет в момент пережигания перпендикулярно нити (дальше это, естественно, нарушится, но нас это не волнует). Отсюда ты можешь найти новую Т.
> 11.11
Я бы просто просуммировал энергии взаимодействия шаров друг с другом. Т.е. 3kq^2/l. Ну и для линии так же.
> 11.12
Я что-то не очень могу проследить ход твоих мыслей, но главное - равенство T - ты использовал. Вот эта картинка должна прояснить остальные уравнения. Спойлер (не читай его!): T = F1 = F2 = F3

Ах да, мне лень доводить решения до конца, я просто подкидываю идеи, которые бы использовал сам. Естественно, они могут быть в корне неверными.
>> No.150409 Reply
>>150308
Ну да, нерастяжимый, но сила натяжения какая-то у него есть же. Я не понимаю, о чем ты. Что за ненулевая правая часть? Вот был шарик В. В начальный момент времени Т+ФС+Ф1+Ф2=0 (все-векторы, конечно). Левую ниточку пережгли. Что и как изменилось для правого шарика, как?
> 11.1
А как это сделать? Ну то есть, вот даже не для этой задачи, а вообще: есть какое-то кол-во заряженных тел. Или не заряженных. Как посчитать потенциальную энергию системы таких тел? Я должен выбрать какое-то "главное" тело, относительно которого я буду считать энергии других тел и потом просуммировать их? Или найти потенциал каждого тела относительно всех других, а из него потенциальную энергию каждого из них, а затем просуммировать? Как ты получил свой ответ?
>> No.150411 Reply
>>150409
> Что за ненулевая правая часть?
Приехали. То есть ты даже не знаешь, что за уравнения ты записываешь? Ты просто делаешь по образцу штоле? Эти системы - это тот самый n-й закон Ньютона, который F=ma. Где F - равнодействующая всех сил, m - масса, a - ускорение. В статической системе a=0 (да-да, именно ma - это твоя правая часть, и далеко не только в этой задаче). В момент пережигания нити система перестает быть статической и приобретает ускорение. И вот это ускорение ты можешь посчитать, если знаешь его направление.
> Я должен выбрать какое-то "главное" тело, относительно которого я буду считать энергии других тел и потом просуммировать их?
Нахрена? Посчитай попарно энергии взаимодействия и сложи их. Не важно относительно чего считать энергию взаимодействия джвух заряженных шаров друг с другом.

P.S. Судя по частоте твоего появления здесь, не особо оно тебе и надо.
>> No.150412 Reply
File: 1389.png
Png, 6.84 KB, 200×163 - Click the image to expand
edit Find source with google Find source with iqdb
1389.png
>>150257
> В обоих задачах
В обеих.
>> No.150451 Reply
>>150411
Чувак, я прекрасно знаю, что это за уравнения и зачем, не гони. Вот есть уравнение для первого тела. После пережигания нити в нем пропадает сила натяжение переженной нити и появляется ускорение. Вот есть уравнение для второго тела. И что с ним происходит после пережигания нити я не знаю. И потому не могу найти ускорение второго тела. Именно это мне и нужно понять: что изменится в уравнении второго тела после пережигания нити.
>> No.150871 Reply
Здравствуй, сонный тред. Нужна помощь. Я учусь в техническом вузе, в котором для галочки нужно изучать 2 года физику (я на первом курсе сейчас). Физику я не сдавал, физику я криво изучал в школе, имея 4, решая задачи в одно-два действия, и к физики особо интереса я не имею (даже раздражение, вспоминая всех своих противных физичек). Итак, мне хотелось бы советов годной литературы, которая бы мне помогла справиться с нынешними проблемами в физике, желательно, чтобы в книге были подробно расписаны задачи, было много разных примеров. В универе имею И.В. Савельева, который понятно пишет, книга быстро читается, но пользы от него мало, т.к., как мне кажется, много воды. Мне советовали Г.С. Ландсберга, но, по-моему, с ним та же ситуация: понятно, вполне интересно, но пользы при решении задач особо никакой.
>> No.151150 Reply
File: tumblr_npwpz8Y17N1s076u4o1_540.jpg
Jpg, 188.76 KB, 540×764 - Click the image to expand
edit Find source with google Find source with iqdb
tumblr_npwpz8Y17N1s076u4o1_540.jpg
Аноны. Видал на хабрахабрах статьи по алгебре/тензорах/матану и прочее. Там все довольно неформально было описано, не точно, много что упущено. Но что понравилось, после прочтения, углубляться в серьезную литературу легче. Но по физике там ничего нету. А меня интересует: динамика решеток, нормальные координаты, описание колебаний в кристаллах. Не подскажете никакие энтрилевельные статьи/материал? Спасибо.
>> No.151438 Reply
Медленно вникаю в электричество, входит туго в том числе потому, что матана не хватает.

Что рекомендуете? Быстро погуглив узнал что есть Фейнмановские лекции, Берклиевский курс, Ландавшиц, Савельвев и Сивухин.
>> No.151453 Reply
>>151438
> Фейнмановские лекции, Берклиевский курс, Ландавшиц, Савельвев и Сивухин.
Из всего этого наиболее хаброобразно написан курс Фейнмана.
>> No.151461 Reply
>>151453
> Из всего этого наиболее хаброобразно написан
Нет-нет, это два разных анона (кажется). Один просил хаброобразно, но теорию кристаллов. А другой просто спросил про учебники электричества, без особых требований к стилю..
>> No.151462 Reply
>>151150
> Не подскажете никакие энтрилевельные статьи/материал? Спасибо.
Сам бы хотел узнать.

Вот чтобы глубоко копать - это Ашкрофт/Мермин, это я знаю. А вот чтобы именно изложение симметрии кристаллов и всего такого, и чтобы доступно... присоединяюсь к вопросу.
>> No.151463 Reply
>>150451
> Вот есть уравнение для второго тела. И что с ним происходит после пережигания нити я не знаю.
Первое тело, начиная двигаться, тянет за собой второе. Полагаю, причина в этом. Можно сказать, что на каждое из двух тел действовала (в статике) сила реакции опоры (этого самого стержня, распиравшего их). Когда 1-й шарик начал двигаться, то равнодействующая сил на него будет такая, что в любом случае он "уйдёт" от этого стержня => снимется эта сила. Ну или можно сказать, что не снимется, а уравновесится импульсом 1-го шарика, начавшего двигаться. Для идеального неупругого стержня это более правильная формулировка...

Вот и появляется ненулевая равнодействущая на 2-й шарик сила... вот и причина ему ускориться.
>> No.151465 Reply
File: 1459507373.jpg
Jpg, 41.57 KB, 447×604 - Click the image to expand
edit Find source with google Find source with iqdb
1459507373.jpg
>>151461
Ты совершенно прав. Я долго думал, "хаброобразно"- это хорошая характеристика?
BTW, Феннман мне понравился, стало понятнее как это работает в целом, но еще задачи решать надо.
>> No.151477 Reply
File: 124291629271.jpg
Jpg, 138.23 KB, 700×600 - Click the image to expand
edit Find source with google Find source with iqdb
124291629271.jpg
>>151462
Собственно я нашел энтрилевел по симметрии. Гугли Шаскольская Кристаллография. Но именно по динамике решеток нихуя пока.
151150-анон
>> No.151888 Reply
Сап, объясните, пожалуйста, чайнику, как учёные определили форму орбиталей атома. Я слышал от Гугла про квантовые числа, но я не понял, какое отношение они имеют именно к форме орбиталей.
>> No.151890 Reply
>>151888
уравнение Шрёдингера, не?
>> No.151906 Reply
>>151888
Форма орбитали - крайне расплывчатое понятие. Плотность вероятности нахождения электрона в какой-то конкретной области пространства находится с помощью уравнения Шрёдингера, например, но это именно что плотность вероятности, а не орбиталь с заданной формой как в классической механике.
>> No.151909 Reply
>>151906
Ну смотрим где плотность вероятности выше, там получается некое облако. Это облако и имеет форму орбитали, не? Орбитали, а не орбиты*.
>> No.151948 Reply
File: 13992604271552xet5.png
Png, 127.87 KB, 566×528 - Click the image to expand
edit Find source with google Find source with iqdb
13992604271552xet5.png
Антоны, подскажите, где в Украине можно приобрести Берклеевский курс физики и Л.-Л., желательно онлайн, без регистрации и смс? Потому что живу в таком захолустье, куда почтовые голуби доходят с трудом.
>> No.151958 Reply
>>151948
> живу в таком захолустье
Где, если не секрет?
>> No.151963 Reply
Привет, анон. Помоги мне с дифракцией. Куча вопросов.
1. Зоны Фринеля. Вот, допустим, смотрим сюда: http://ens.tpu.ru/POSOBIE_FIS_KUSN/%D0%9A%D0%BE%D0%BB%D0%B5%D0%B1%D0%B[...]2.htm
Или ещё куда-угодно, где поясняют за дифракцию. Почему складывают колебания от целых ЗОН и говорят, что разность хода между ними равна лямбда\2? Ведь колебания испускаются не как-то целой зоной, а каждой точкой в пределах этой зоны. И если разность хода между центром первой зоны и началом второй, или концом первой и концом второй равна половине волны, то разность хода между серединой 1 зоны и второй может быть какой угодно. Почему замена всех точек на фронте волны отдельными зонами считается правильной? Написанное в конце
> > Правомерность деления волнового фронта на зоны Френеля подтверждена экспериментально.
Меня не устраивает. Не люблю принимать что-то на веру.
2. http://ens.tpu.ru/POSOBIE_FIS_KUSN/%CA%EE%EB%E5%E1%E0%ED%E8%FF%20%E8%2[...]4.htm
Почему вообще берется какая-то разность хода? Ведь, например, ABOD - параллелограмм. И пути, проходимые лучами света, равны. Что вообще в этом случае понимается под зонами Фринеля?
>> No.151966 Reply
>>151958
> Где, если не секрет?
> в Украине
Если у тебя есть конкретные предложения касательно продажи, к примеру, через Новую почту, так и написал бы.
>> No.151968 Reply
>>151966
Мне просто интересно, я ведь тоже с Украины, и тоже, как студент-биолог понимаю, насколько мы в жопе с наукой, и поэтому придется строить трактор, который у меня плохо то получается делать, с моей то социализацией.
>> No.151969 Reply
>>151968
Я, как потенциальный экономист, тоже понимаю бесперспективность пребывания в Украине. Работу я-то найду, даже очень неплохую, но надолго меня не хватит. Куда интересней трудиться в области точных наук, тем более за рубежом. Пока не просрал все полимеры, заведу трактор и укачу в закат. Пусть только эти свинорылые политиканы договорятся о безвизовом режиме, дабы проблем с её оформлением не возникло.
>> No.151970 Reply
>>151948
А банальное djvu тебя не устроит? Надо именно в бумаге?
>> No.151972 Reply
>>142910
Одно слово — диффузия.
>> No.151978 Reply
>>151970
В электронном формате у меня уже давно есть. На бумаге куда сподручней ставить заметки, особенно, когда в книге опечатки. Все не было бы так прискорбно, если бы мой знакомый профессор не потерял 4 и 5 том Берклеевского курса и несколько томом Л.-Л. Поэтому, я вынужден найти их все и приобрести для личного пользования. Тем паче, я склонен считать, что конкретно эти книги всегда в хозяйстве пригодятся.
>> No.151981 Reply
Сап физач. Есть один ребенок 10-ти лет нет, не я, хочу рассказать ему о физике и как-то заинтересовать. С чего бы начать ему рассказывать и показывать? Можно ли взять Л-Л самому и описывать на доступном языке основы физики?
>> No.151982 Reply
>>151981
Очевидный Рэнделл Монро жэ. Он сам по манере повествования ребёнок, так что 10летнему п-ну понравится.
>> No.151983 Reply
>>151981
Я сам начал с книг Перельмана в свое время. Нравились его анализы известных мне прочитанных романов. Хотя до этого еще читал всякие энциклопедии по физике, особенно с полетами, ракетами там связанное.
>> No.151984 Reply
File: 14124155487062.jpg
Jpg, 57.89 KB, 600×615 - Click the image to expand
edit Find source with google Find source with iqdb
14124155487062.jpg
>>151981
Не в обиду сказано, но ты вообще видел(а) содержание Л.-Л.? Ты бы ему ещё ЖЭТФ/УФН предложил(а) на ночь почитать, вместо сказок. Сомневаюсь, что твой знакомый ребёнок - будущий Померанчук но всяко бывает.

Возьми, для начала, очевидно, "Занимательную физику" Перельмана, в двух томах. Также, следуешь по ссылке http://publ.lib.ru/ARCHIVES/B/''Bibliotechka''Kvant''/''Bibliotechka_''Kvant''.html и смотришь книги, тематика которых тебя привлекает. Если желаешь наглядности, я бы советовал "Опыты в домашней лаборатории", чтобы на конкретных примерах пояснить всю мощь прикладных аспектов физики.

Помимо этого, существует ещё пара журналов для подростков. Обрати внимание, в частности, на такие:
http://potential.org.ru/
http://kvantik.com/
Можно качать электронную версию и давать почитать. Непонятные места, очевидно, пояснять придется самостоятельно.

В случае, если он всерьез заинтересуется физикой, можешь переходить на Ландсберга/Перышкина/Мякишева, чтобы сформировать у него представление о базовых физических понятиях в пределах строгости, в которых это позволяет школьное изложение.
>> No.151985 Reply
File: Cookies(1).png
Png, 1066.76 KB, 1000×932 - Click the image to expand
edit Find source with google Find source with iqdb
Cookies(1).png
>>151984
>>151983
>>151982
Спасибо всем, с меня печенье.
>> No.151986 Reply
>>151985
Извини, на "Библиотечку Квант" ссылка не работает. Вот другая. http://www.math.ru/lib/ser/bmkvant
>> No.151994 Reply
File: 14374281904200.jpg
Jpg, 33.79 KB, 419×604 - Click the image to expand
edit Find source with google Find source with iqdb
14374281904200.jpg
Антоны, сведущие в немецком, отзовитесь. Читал давеча об одном физике, конкретней: Wolfgang Nolting. Вот статья на вики: %% https://de.wikipedia.org/wiki/Wolfgang_Nolting_%28Physiker%29 %%

Больше всего меня, естественно, заинтересовала вот эта фраза: "Bekannt geworden ist Wolfgang Nolting unter anderem durch seine Lehrbuchreihe Grundkurs: Theoretische Physik, welche mittlerweile den Rang eines Standardwerkes in der Physikerausbildung erlangt hat." Судя по этим словам, он написал аналог (если, конечно, можно сравнивать его книги с этим) Л.-Л. для немецких студентов.

Есть ли у кого-то доступ к электронной версии его книг? Все мои поисковые запросы выдают ссылки только на урезанные версии. Если ткнете меня носом, буду благодарен.
>> No.151995 Reply
>> No.151998 Reply
>>151995
Спасибо же.
>> No.152003 Reply
Антоны, есть у кого ссылки на источники, в которых подробно описывают опыты Хофштадтера? Смотрел в "Экспериментальной ядерной физике" Мухина - ничего нет. В книге Мякишева "Силы в природе" только небольшое упоминание. В Википедии ссылка только на Нобелевскую лекцию.
>> No.152034 Reply
Привет физач, хочу окунуться в мир квантовой механике, посоветуй, с чего стоит начать?
>> No.152037 Reply
>>152034
Начни со школьного курса физики. Потом - университетский. После этого можешь окунаться.
Если желаешь научпопа - качай, например, Гинзбург, Андрюшин "Сверхпроводимость", Фейнман - "КЭД Странная теория света и вещества". Можешь так же заглянуть в "Дюжину лекций" Фейнмана и "Физика для всех. Фотоны и ядра" Китайгородского. Там обо всем понемногу.
>> No.152047 Reply
>>152037
Труды господ Л.-Л сгодятся для > Начни со школьного курса физики. Потом - университетский. ?
>> No.152049 Reply
>>152047
Л.-Л. - курс теорфизики. Это далеко не начальный уровень. Сначала читаешь Ландсберга/Мякишева/Перышкина, потом Берклеевский курс/Сивухина/Фейнмана/Савельева. Когда покончишь с этим, можешь приступать к Л.-Л., если действительно хорошо усвоил материал.
>> No.152050 Reply
File: 01866D9F-1A51-414C-8A72-591E275D0505.jpg
Jpg, 162.34 KB, 850×1219 - Click the image to expand
edit Find source with google Find source with iqdb
01866D9F-1A51-414C-8A72-591E275D0505.jpg
>>152049
Спасибо. Держи няшку.
>> No.152065 Reply
Аноны, у меня, похоже, кризис. Объясните мне кое-что, иначе голова взорвется. Рассмотрим СО, связанную с Землей. С некоторой долей допущения, она - ИСО. Но ведь все тела на поверхности земли движутся в поле её тяготения с ускорением g.

Первый закон Ньютона гласит - Существуют такие системы отсчёта, называемые инерциальными, относительно которых материальные точки, когда на них не действуют никакие силы (или действуют силы взаимно уравновешенные), находятся в состоянии покоя или равномерного прямолинейного движения. Получается, если есть ускорение - есть сила, в данном случае, сила притяжения Земли. Мы ней что, пренебрегаем, в угоду тому, чтобы назвать Землю ИСО?

И ещё один вопрос. Рассмотрим СО , связанную с материальной точкой, движущейся в поле тяготения Земли. Это ИСО или НИСО и почему?

Подробней, пожалуйста.
>> No.152078 Reply
>>152076
Для начала линал и ТФКП.
>>152065
> когда на них не действуют никакие силы
А сила тяжести - это сила.
Вообще, ИСО - это абстракция, с которой удобно работать, не более. Если мы учли все внешние факторы как какие-то виртуальные силы (или положили на них болт потому что можем), то у нас ИСО. Например система, связанная с вращающимся кольцом вполне себе инерциальна когда мы вводим виртуальные центробежную силу и силу Кориолиса.
А в ОТО никаких инерциальных систем нет, все системы равноправны.
>> No.152081 Reply
>>152078
Спасибо, анон, касательно того, что благодаря Эйнштейну был сделал вывод об отсутствии в природе абсолютной системы отсчета, я был осведомлен. Но мой вопрос, скорее, ближе к школьной физике. Мне объяснили, что если, с некоторой долей допущения, считать Землю ИСО, равнодействующая сил и вправду равна нулю. Ведь есть сила притяжения и сила реакции опоры.
>> No.152131 Reply
Доброе утро, аноны. Мне тут задали по инфе такое задание: вроде бы легкое, а все не получается адекватно заставить работать. Может, кто сталкивался с подобным. Надо смоделировать движение точки в поле тяжести, учитывая отскакивания от стен комнаты (все в 2д) и от одной наклонной плоскости. Вроде все легко, а энергия теряется. Попытался задавать только направление отскока, а скорость высчитавать по формуле закона сохранения энергии, а он адекватно её не считает, вылетает с ошибкой через некоторое время, мол, я идиот, корень из отрицательного числа беру. Но это ладно, может, со знаками напутал (у меня система координат немного кривая). Заставил показывать полную энергию, а она, оказывается, скачет от 0 до 3000. Кинетическая, кстати, более менее сохраняется (её максимум падает на 1 примерно за 10 скачков. Начальная 30.) Буду пытаться дальше, конечно, но дедлайн во вторник, а эту прогу уже месяц как надо сдать. Поэтому прошу совета, как заставить сохранять энергию. Кому интересно, все это прогается на ФОРТРАНЕ. 2016 год. Сначала делал отрисовку просто по циклу, но недавно переписал уже по параметру T с шагом 0.001, задавая при отскоке функцию, по которой будет летать.
>> No.152132 Reply
>>152131
И что мешает просто в момент столкновения разложить вектор скорости на составляющие: параллельную стене и перпендикулярную, и в перпендикулярной тупо поменять знак?
>> No.152133 Reply
>>152132
Делал так, но, говорю же, энергия со временем заметно теряется.
>> No.152135 Reply
>>152133
Тогда ему нужен ограничитель, который останавливает вычисление в момент, когда Е=0, чтобы расчет не уходил в минус.
>> No.152136 Reply
>>152133
Как она там может теряться? Скорость-то остается одной и той же. Вероятно, в "момент" столкновения у тебя происходит что-то еще. Останови время, пересчитай скорость, запусти время снова.
>> No.152138 Reply
>>152136
Хм, действительно, я так подумал и понял, что при столкновении я для "точности" присваиваю точке координату стены. Отсюда и изменение энергии. Похоже, придётся чаще проверять касание со стеной и не менять координаты. Но все равно спасибо за наводку.
>> No.152319 Reply
Доброаноны, помогите, как можно найти массу планеты если дано два радиуса(полярный и экваториальный) и период вращения вокруг своей оси?
>> No.152536 Reply
>>137425
Аноны, что можно почитать про статистики Ферми-Дирака и Бозе-Эйнштейна? Ну и про распределение Гиббса. Только без очень сложного матана желательно, чтоб первокурснику было понятно.
>> No.152546 Reply
File: A9JFHKSBs7s-(1).jpg
Jpg, 77.86 KB, 604×453 - Click the image to expand
edit Find source with google Find source with iqdb
A9JFHKSBs7s-(1).jpg
>>152536
Матвеев молекулярная физика. Там первая часть про статистический подход. В ней про распределение Максвелла\Гиббса\Больцмана. В общих чертах про Ферми-Дирака в моем издании упоминалось (подробности надо искать в квантмехе, первокурснику не надо это). Матан не сложный, но психологически отталкивают термины. Именно статистический подход подробнее описан в Матвееве лучше чем в Сивухине. Только ради Христа читай последовательно, начинай с первых параграфов и не скипай ничего.
>> No.152550 Reply
>>152546
Хорошо, спасибо, попробую.
>> No.152729 Reply
File: IMAG0054.jpg
Jpg, 334.77 KB, 2397×405 - Click the image to expand
edit Find source with google Find source with iqdb
IMAG0054.jpg
Всем добрый.
Есть одна задача по теормеху, а я не знаю, с чего начать.
Ландау и Лифшиц не помогли, доброанон, подскажи, где посмотреть похожие или с чего начать, а то как-то совсем пусто.

P.S. Так-то нашел похожее в Мещерском, но у меня все равно по-другому.
http://bambookes.ru/stuff/reshenie_zadach/teoreticheskaja_mekhanika/tochka_massy_kg_dvizhetsja_po_gladkoj_poverkhnosti_kruglogo_konusa_ugol_rastvora_kotorogo_90_pod_vlijaniem_sily_ottalkivanija_ot_vershiny/11-1-0-3542
>> No.153309 Reply
File: Cromartie.High.School-18-Puutan.Gets.Fired.[NAG-He.jpg
Jpg, 57.18 KB, 720×544 - Click the image to expand
edit Find source with google Find source with iqdb
Cromartie.High.School-18-Puutan.Gets.Fired.[NAG-He.jpg
Что-то мне подсказывает что тред мертв, но я попробую спросить.
Анон, поясни за интерференционный метод измерения отклонения от параллельности.
Суть - к плоскости прикладывается стекло-эталон, оно может быть как и цилиндрическое, высотой под 6 см так и меньших размеров,и с верхней части этого цилиндра просматриваются на этой плоскости интерференционные полосы.
Вопрос - почему они там появляться, от чего 1 полосу считают за 3 мкм отклонения от параллельности и почему я должен доверять этому методу.
Спасибо.
>> No.154376 Reply
Катал на егэ, поступил в топчик вуз, ничего не понимаю. Писали лабу. Когда другие уже сдавали тетради, я пытался врубиться в формулы из методички (сумма ряда, дифференцирование), я вообще не понимал, что куда подставлять и что от меня требуют. В итоге сдал пустую тетрадь, лол. Можете обоссывать меня, но это правда единственный предмет, который я не понимаю, а с матаном и инженеркой все заебись. Как быстро наверстать знаний по физике, если в гуманитарной школе ничего не давали? Какие-то пособия для совсем уж даунов существуют? Чтоб там расписывалось на примерах с машинками и грузиками, а не кракозябрами ебаными.
>> No.154378 Reply
>>154376
Возьми в библиотеке методичку и втыкай. Гугли каждое непонятное слово.
>> No.154450 Reply
Привет, тред, вот наш профессор по термеху говорит, мол, аксиома инерции, на самом деле, неверна. Хочет, чтобы мы сами догадались. Так вот, почему она неверна-то?
>> No.154452 Reply
>>154450
Можешь формулировку запостить?
>> No.154455 Reply
>>154376
> с матаном все заебись
> врубиться в формулы из методички (сумма ряда, дифференцирование)
> Чтоб там расписывалось на примерах с машинками и грузиками, а не кракозябрами ебаными
/0
Иди матан учи, в суммирование рядов он не втыкает...
>> No.154484 Reply
Анон, трудно ли поступить в аспирантуру? И надо ли? Вопрос возник из за военной кафедры, как бы если все равно пойду в аспирантуру, зачем мне терять месяц жизни? скорее всего, одна из кафедр мфти
>> No.154562 Reply
>>154376
> я пытался врубиться в формулы из методички (сумма ряда, дифференцирование), я вообще не понимал, что куда подставлять и что от меня требуют
> с матаном ... все заебись
Does not compute.
>> No.154563 Reply
>>154376
Алсо,
> Катал на егэ
> в гуманитарной школе ничего не давали
Сам себе буратино.
>> No.155184 Reply
File: AbQnJdc66os.jpg
Jpg, 108.33 KB, 640×480
edit Find source with google Find source with iqdb
AbQnJdc66os.jpg
File: aUtJiYHH2tI.jpg
Jpg, 35.96 KB, 640×480
edit Find source with google Find source with iqdb
aUtJiYHH2tI.jpg

Доброго времени суток доброанон.
Такие дела - начался второй курс "Колледжа", на котором пошел славный специалитет большими кусками.
Так, увы, получилось, что проболел я начальную тему такой радости, как Теоретические Основый Электротехники.
Теперь нихера не понимаю что происходит, а понимать надо, ибо дальше будет сложнее.
Собственно, дали нам такое задание, и мне очень нужна помощь (разъяснения, методички, хорошие книжки), как это решать.
К преподу, увы, не подойти, ибо злая мегера.
Книги найти не могу - хз куда копать вообще.
>> No.155210 Reply
File: 1465644568002.jpg
Jpg, 65.82 KB, 604×452 - Click the image to expand
edit Find source with google Find source with iqdb
1465644568002.jpg
>>137425
Возникла твёрдая необходимость вспомнить всю физику. Её я в школе знал на твёрдую четвёрку, часто и пятёрки были - в принципе этот предмет мне всегда нравился. Нужно восстановить в памяти весь курс, и тоже нужна книга типа популярной физики с задачками, чтобы разжевано было и написано так, чтобы я понимал суть, а не заучивал как макака.
Ну и посоветуйте отдельно по электродинамике подобного, чтобы отдельно было разжевано. Наперёд спасибо.
>> No.155335 Reply
Объясните почему у электрона в сравнении с протоном и нейтроном такой малый вес? Это обусловлено зарядом? Нейтрон ведь не имеет заряда но по весу близок к протону.
>> No.155341 Reply
>>155335
Это обусловлено его природой. Так же как величина скорости света обусловлена природой пространства.
>> No.155365 Reply
>>155184
Бакалов "Основы теории цепей"
>>155210
Сасскинда попробуй почитать
>> No.155374 Reply
File: 1273411798941.gif
Gif, 32.95 KB, 99×99 - Click the image to expand
edit Find source with google Find source with iqdb
1273411798941.gif
>>155184
Л.А. Бессонов Теоретические основы электротехники . Электрические цепи. 28 стр, Свойства линейных электрических цепей и методика их расчета . Тебе какимто особым методом это решить нужно или без разницы ?
>> No.155375 Reply
>>155210
попробуй Савельева почитать наверно , но это вузовская программа , но там основы все охвачены вроде , 3 тома , механика в 1ом , во 2ом электричество , 3том квантовые штуки и теория атома , как то так .
>> No.155676 Reply
анон, объясни за тихий разряд. почему он тихий? откуда у тока вообще звук берется? конкретно про звук нигде не нашел. при озонировании используют тихий разряд.
>> No.155821 Reply
File: 1340102895121.gif
Gif, 488.39 KB, 500×281 - Click the image to expand
edit Find source with google Find source with iqdb
1340102895121.gif
Посоны, вопрос. Задача трех тел является стохастической верно? Если бы не являлась, ее бы все решали численно и в хуй бы не дули, да ведь?
>> No.155827 Reply
Достаточен ли курс теоретической физики Ландау для последующего полного погружения в современную физики, или есть альтернативы получше? Интересует литература, заточенная конкретно под физиков: после первых двух курсов в своем универе окончательно убедился, что ни черта тут не учат, а посему хочу наверстать упущенное. Ей богу, аж горит: что будучи школяром приходилось заниматься одному, что теперь, поступив на чисто физика, снвоа приходится все искать самому. За что, курва?
>> No.155832 Reply
>>155827
Ты родился в России. За это. Тут науку прекратили финансировать в семидесятые.
Первый том ЛЛ очень сильно ругают, механику нужно учить по другим книгам. Ну и в целом у Ландау была неприязнь к математике сложнее первого курса, так что после ЛЛ придётся доучиваться. Есть также мнение, что общие курсы физики - зло, надо читать специализированные книги по каждому разделу. Английский знаешь? Если нет, срочно учи. На русском пишет http://igorivanov.blogspot.ru/?m=1 , у него есть списки книг, погугли по блогу.
>> No.155837 Reply
>>155832
> Ты родился в России
Забавно, но я учусь в Шотландии в далеко не самом последнем заведении. После твоих слов о России у меня вообще начинает складываться такое впечатление, что нормально преподают разве что в самых топовых ВУЗах. За ссылку спасибо, ознакомлюсь.
>> No.155848 Reply
>>155837
Биография видных ученых публична. Заводишь отдельный файлик, гуглишь биографии ученых помоложе, складываешь их в файлик, считаешь статистику. Делаешь выводы, где учат, а где за отсидку фантик дают. Наука есть и в этой стране, только надо места знать, где она живет.
>> No.155850 Reply
>>155848
Не посоветуешь тогда куда после школы идти? Хочу быть компьютер-саентистом
>> No.155857 Reply
>>155850
> Хочу быть компьютер-саентистом
А таки позвольте спросить: зачем? Все, что могут делать ИТшники, математики и физики делают как минимум не хуже (особенно математики). С технической стороной вопроса (архитекутра и т.д.) можно без всяких запилов ознакомиться самому.
В общем, подумай хорошенько еще раз.
>> No.155860 Reply
>>155857
Просто я тогда не знаю что дальше делать. Идти куда-то нужно, а я не знаю куда. Самостоятельно начал заниматься, но 11 класс же, нужно куда-то поступать.
>> No.156524 Reply
Бамп
>> No.156619 Reply
Bump
>> No.156620 Reply
Крайне боевая
>> No.156621 Reply
Напоследок вечеру входящему
>> No.156676 Reply
Аноны, помогите с выбором кафедры. В общем так, математику я не очень люблю, но отлично зашла теория вероятности и неплохо аналит. Ознакомился с теорией групп, тоже понравилась. С физикой отношения хорошие, но вот теормех, например, не зашёл. Только кватернионы и Лагранжиан понравились, остальное в топку. Из последнего зашли, например, объяснения всяких вещей вроде теплопроводности или диамагнетизма, рассматривая молекулы как механические частицы (лол, лет на сто инфа устарела, но всё же). Волны - что то не то. Информатика нравится, но заниматься ею не хочу.
Как я заметил, названия кафедр делятся на несколько видов: теория...., проблемы...., и остальное, методы, например. Кто-то сказал, что если в названии есть слово теория (струн, например), то это чисто матан и о физике там можно забыть, что меня не очень привлекает.
Так вот, открылась у нас "кафедра теории струн, квантовая теория поля и матфиз". Шанс поступления туда стремится к нулю, но всё же: мне с такими статами подойдёт там учеба? И что насчет физики элементарных частиц и прочего (ИТЭФ)? И да, если можете, подскажите, на какие ещё можно кафедры интересные поступать?
>> No.156679 Reply
>>156676
У тебя необычные представления о физике. Может быть, позже соберусь с духом и напишу тебе ответ.
>> No.156680 Reply
>>156676
> лол, лет на сто инфа устарела, но всё же
Ничего не устарело. Просто ты еще не всосал суть физики. Анон когда ответит, я может чего добавлю. Сразу только скажу, что лучше не лезть в теории струн и элементарные частицы. Выбирай кафедру беря в расчет то, есть ли хорошая лаборатория в институте, которая этим занимается (по теории струн скорее всего нет).
>> No.156696 Reply
File: [-__-]-Moyashimon-03-[BD-1080p-x264-8-bit-FLAC]-[E.jpg
Jpg, 128.73 KB, 1920×1080 - Click the image to expand
edit Find source with google Find source with iqdb
[-__-]-Moyashimon-03-[BD-1080p-x264-8-bit-FLAC]-[E.jpg
Какие есть свидетельства, указывающие на то, что другие кластеры галактик состоят именно из вещества, а не из антивещества? Как бы мы видели различия, если бы они состояли из антивещества?
>> No.156709 Reply
>>156676
Что насчёт физики твёрдого тела?
>> No.156712 Reply
>>156696
Потому что пространство между ними наполнено газом. Хоть этот газ и очень разряжен, но если бы он реагировал с антивеществом, то это все равно было бы заметно.
>> No.156714 Reply
>>156712
Насколько заметно? Где можно прочитать про это? Насколько я это вижу, большая часть вещества в принципе никогда не покидает пределов кластера, а то, которое покидает, может легко пересечь войд и ничего там не встретить, а если и встретит, то получится гамма-квант, который наблюдатель никак не сможет выделить из фона.
>> No.156737 Reply
>>156709
Собираюсь этим заниматься. Третий курс. Есть интерес, да и рядом с университетом есть институт физики занимающийся этим. Спрашивай свои вопросы.
>> No.156820 Reply
Закончил с экзаменом, теперь могу отписаться. Звиняйте, что так поздно.
>>156709
>>156737
это писали два разных человека

В общем, почитал я про это немного, и показалось, что там много химии, которой я не переношу. Хотя сам сейчас на спектроскопии, и не знаю, что делать. И что там не так много теории, а гораздо больше практики.
К слову, так ли там много химии? И тот же вопрос про спектроскопию.

На выходных буду много читать про разные кафедры. Будут вопросы - спрошу.

>>156679
Как я понял, вам не понравилось, что я слишком отделяю физику от математики. Просто у нас на лекциях и семинарах часто говорят: "Ну, мы не математики, угадываем ответ и доказываем его", или "Вот ответ, давайте угадаешь знак", или "Вот два ответа. Один невозможен." Или эти приблизительные вычисления производных и интегралов в одну строку. Поэтому выражение "математическая строгость" как то с физикой у меня вообще не ассоциируется, да и не люблю я ее. И если теория струн будет такой же строгой, как этот матанализ, например, я просто консиднусь.
>> No.156821 Reply
>>156820
> Ну, мы не математики, угадываем ответ и доказываем его
Угадывать ответ можно, даже если ты — математик. Хотя это совсем не их стиль.
> показалось, что там много химии
Та «химия» скорее является физикой, чем химией.
Почему не переносишь, кстати?
>> No.156826 Reply
>>156821
Химия у меня была в школе и на первом курсе. За все это время я не понял методов, с помощью которых она работает. Для меня она - набор экспериментальных данных, которые можно только выучить.
>> No.156827 Reply
>>156820
> что там много химии
Она не та, что на уроках химии. Там скорее о взаимодействии электронов и ядер. Ну и о взаимодействии атомов на квантовом уровне. Квантовая химия по сути. К той химии, к которой ты привык никакого отношения почти, но тоже химия.
> на спектроскопии
Исследование структуры возможно тебя заинтересует. Это экспериментальная физика, так что необходимо узнать о наличии установок в лабах. Ну и сама спектроскопию тоже интересна, но с химией почти не связана. Оч много используется из атомной физики (электронные структуры, квантования и прочее). Еще оптика будет нужна, хотя бы общий курс.
> И что там не так много теории
Теория есть вполне в ФТТ. Погугли например ab initio calculation, DFT.
>> No.156858 Reply
>>156820
> Как я понял, вам не понравилось, что я слишком отделяю физику от математики. Просто у нас на лекциях и семинарах часто говорят: "Ну, мы не математики, угадываем ответ и доказываем его", или "Вот ответ, давайте угадаешь знак", или "Вот два ответа. Один невозможен." Или эти приблизительные вычисления производных и интегралов в одну строку. Поэтому выражение "математическая строгость" как то с физикой у меня вообще не ассоциируется, да и не люблю я ее. И если теория струн будет такой же строгой, как этот матанализ, например, я просто консиднусь.

Постой-ка, но ты описал, как я занимаюсь математикой (я математик)! Хотя математики разные бывают. Но математика - это не только как рассказывают матан/линал на первых курсах универов.
>> No.156859 Reply
>>156858
У математиков же принято осмыслять математическую деятельность, не?
>> No.156866 Reply
>>156858
Странный ты математик.
>> No.156869 Reply
>>156858
Может, ты и прав. Мы учили теорвер, и у нас он был вообще не строгий. Да и диффуры - набор методов, которыми пользуются, используя то, что мы строго получили на матанализе с нуля.

>>156827
Спасибо, понял примерно, в каком направлении копать.
>> No.156871 Reply
File: IMG_20161226_233245.jpg
Jpg, 2003.90 KB, 3920×2204 - Click the image to expand
edit Find source with google Find source with iqdb
IMG_20161226_233245.jpg
>>156869
Олсо, вот, кстати, пример достаточно физического метода решения.
>> No.156959 Reply
Хватит заниматься фигней, я тут вам принес РЕАЛЬНУЮ ЗАДАЧКУ.
В общем, смотрел видосики и наткнулся на такую тему: ребята бьют по открытому горлышку бутылки (стеклянной) сверху-вниз, в результате чего выбивается дно этой самой бутылки. Можете разобрать мне физику этого процесса? Почему так получается. Ведь есть силы связей, все такое, и в итоге, весь удар будет приходится очевидно на горлышко, а для нижней части бутылки, около дна, воздействие будет очень слабое. Для того, чтобы воздействовать на дно бутылки нужна сила, которая скорее раскрошит горлышко бутылки оно оставалось целым. Единственный вариант тогда - этим ударом нагнать воздуха в бутылку, чтобы повысить давление. Но и это тоже бред, т.к. человеческая рука + бутылка это ни разу не герметичный насос. Да и тогда скорее бы выбивало стенки бутылки т.к. они тоньше донышка.
>> No.156969 Reply
>>156959
Очевидно, что дело в создаваемом гидравлическом ударе. В том видео, что я нагуглил, контакта горлышка и бьющей руки не происходит в принципе, удар идёт во вторую руку, держащую бутылку за горлышко.
Разрушение при этом происходит не в самом дне, а в области соединения дна и стенок, видимо, там наименее прочная часть из-за специфики литья.
>> No.156971 Reply
>>156969
Поясни. На википедии написано, что гидравлический удар, это когда резко увеличивается или уменьшается давление из-за изменения скорости потока жидкости. Но в бутылке вода не движется совсем. Да и нельзя же там давление как-то повысить, рука не гермитезирует бутылку же.
Ну то, что разрушение не в самом дне это понятно, не совсем так выразился.
>> No.156975 Reply
>>156971
Нуок. Давление там как раз таки должно повышаться. Оно нагнетается с высокой скоростью и даже при неполной герметичности будет увеличиваться. При этом, поскольку мы говорим не о квазистатичном состоянии, давление что в месте стыка с ладонью, что в бутылке должно бы распределяться неравномерно. Ну представь себе поток молекул воздуха/воды, которые нагнетаются в определённом направлении. Прежде чем их импульс распределится равномерно по всем направлениям (создав равномерное давление на стенки) некоторое краткое время в направлении удара импульс будет выше - то есть сила удара придётся преимущественно в дно. На таких же основаниях воздух будет лишь частично выходить через щели между руками в финальной стадии удара, большая его часть уйдёт в бутылку.
Конечно, всё это нужно прикидывать с формулами, но сама физика процесса мне видится такой.
>> No.156981 Reply
вроде бы про вашу бутылку было видео обоснование на том же ютубике
>> No.157183 Reply
File: IMG_0090.JPG
Jpg, 2298.54 KB, 4608×3456 - Click the image to expand
edit Find source with google Find source with iqdb
IMG_0090.JPG
>>137425
Аноны, объясните мне, пожалуйста, с какого хрена у меня грифели от карандаша притягиваются к неодимовому магниту? Графит диамагнетик же, отталкиваться должен. А у меня еще и по силовым линиям ориентируется. Что я делаю не так? Может, производители добавляют туда еще что-то кроме графита? Почему в видосиках на ютубе у них получается диамагнитная левитация с графитом, а у меня нет?
>> No.157185 Reply
>>157183
Погуглил. Некоторые грифели для мехкарандашей делают не из графита, а из полимеров разных. Возможно твой как раз из таких.
>> No.157190 Reply
>>157185
Пробовал несколько карандашей разных производителей, не только грифели от механических. Один и тот же результат. Купил новые грифели. На упаковке написано, что в составе пластик и графит. Пластик тоже диамагнетик. То же самое.
>> No.157195 Reply
>>157190
>>157183
Наверно, там, где закупаешься, все грифели просроченными продаются.
Я бы с магнитом и грифелем в Роспотреб бы пришёл и показал бы им всё это, дело ведь серьёзное.
>> No.157196 Reply
>>157195
Но ведь просроченный грифель парамагнетиком все равно не станет, разве нет?
>> No.157197 Reply
>>157196
Если обнимать Его, разговаривать с Ним, шептать Ему о Замыслах Божьих - не станет.
Ухоженный и согретый Грифель никогда не поступит так, как не будет угодно пригревшему Его.
>> No.157208 Reply
>>157190
> Пластик тоже диамагнетик
Спорно. Пластики разные бывают. Но энивей, моя догадка жалко выглядит, не знаю в чем дело.
>> No.157211 Reply
>>157183
Проверил свои грифели, одни (stabilo) тоже притягиваются, другие (нонейм) не реагируют. Интересно.
>> No.157213 Reply
Я погуглил получше по графит. Во-первых, он может содержать ферромагнитные примеси, во-вторых, при наличии дефектов кристаллической структуры или при облучении протонами или другими частицами тоже может проявлять ферромагнитные свойства. Не знаю правда, насколько сильный вклад это все может вносить, но другого объяснения не нашел.
>> No.157216 Reply
>>157213
> дефектов кристаллической структуры
Вот это прям стопроцентов. Я бы на этом мнении остановился.
>> No.157217 Reply
>>157216
Интересно, выходит можно как-то "взбить" графит, чтобы он стал магнитом?
Можно ли так же сделать с неодимовым сплавом для того чтобы неодимовый магнит стал еще сильнее?
>> No.157218 Reply
File: [Beatrice-Raws]-Kizumonogatari-2-[8.58].jpg
Jpg, 97.15 KB, 1920×816 - Click the image to expand
edit Find source with google Find source with iqdb
[Beatrice-Raws]-Kizumonogatari-2-[8.58].jpg
>>157217
С ферромагнетиками такое не прокатит, я почти уверен. Мне кажется просто там домены в графите ломаются и поэтому они теряют диамагнетизм (конечно я нихуя не знаю). Хз, на след неделе я поговорю с чуваком знающим в институте. Если что-нибудь интересное скажет, я сюда напишу на счет этого.
>> No.157219 Reply
>>157218
> домены в графите ломаются
Не понял, какие домены? Домены есть только в ферромагнетиках. Раз уж графит становится магнитным, то наоборот, домены формируются вокруг дефектов кристалла. Или разделяются ими.
>> No.157220 Reply
>>157219
Да ладно, забей, я хуйню сморозил. Не знаю почему от дефектов появляется ферромагнетизм. Скинь статью, где ты это прочитал, если можешь, плиз?
>> No.157221 Reply
>>157220
Предположение о дефектах кристалла не мое, но я нагуглил вот это https://arxiv.org/abs/0910.2130
Буду сам разбираться.
>> No.157222 Reply
>>157221
Счас почитаю, мне тоже интересно.
>> No.157226 Reply
>>157221
Да, я эту же статью имел в виду, когда говорил.

>>157213
>>157183
>> No.158178 Reply
File: Bfk08jo1WVw.jpg
Jpg, 79.36 KB, 700×700 - Click the image to expand
edit Find source with google Find source with iqdb
Bfk08jo1WVw.jpg
Что по квантмеху почитать, кроме Ландау? Хотет про постулаты, понятия и общую концепцию, что и как работает.
>> No.158179 Reply
>>158178
В поздних томах "Фейнмановских лекций по физике" нет ничего подходящего?
>> No.158180 Reply
>>158179
Точн, гляну там, совсем про них забыл. Спасибо, анон.
>> No.158611 Reply
Анон, а вот смотри. В домах же предохранители это магниты, которые при слишком больших напряжениях разрывают цепь, так? А почему вместо них не используют охуенно большие катушки индуктивности? Ток идет, все хорошо, а тут хуяк, огромный скачок напряжения, ща все распидорасит, но не-а, катушка индуцируется и сама гасит весь этот скачок. Никаких проблем, никаких разрывов. Почему так не делают?
>> No.158612 Reply
>> No.158615 Reply
>>158611
Ток в цепях внезапно переменный. Он не пойдет также легко через охуенно большую катушку.
>> No.158649 Reply
Решил попробовать сделать голограмму. Что можете посоветовать? Какие подводные камни? Стоит ли делать фотоэмульсию самому? Можно ли проявить её без лазера?

http://www.media-security.ru/akilov/Handmade%20holo.htm
>> No.158713 Reply
File: [Coalgirls]_Fullmetal_Alchemist_Brotherhood_41_(12.jpg
Jpg, 99.33 KB, 1280×720 - Click the image to expand
edit Find source with google Find source with iqdb
[Coalgirls]_Fullmetal_Alchemist_Brotherhood_41_(12.jpg
Няши, ткните меня носом в публикации на тему экспериментов с поведением античастиц под действием гравитации. А то у меня тут ебанутая идея появилась, надо бы ей научным методом вломить.
>> No.158715 Reply
>>158713
Нашел вот это http://alpha.web.cern.ch/node/248 но по ним гравитационная масса антипротона лежит в диапазоне от -65 до 110 масс протона. Неужели не сделали ничего точнее?
>> No.158718 Reply
Прошу прощения за идиотский вопрос, но существуют ли какой-нибудь сжатый курс элементарной физики с минимумом воды? 1600 страниц Ландсберга пугают.
>> No.158782 Reply
Аноны, поясните некоторые вопросы:
1. Почему если соединить проводником 2 заряженных тела их потенциал станет равным?
2. Как прочувствовать нутром напряжение? К примеру сила тока - это сколько электронов проходят через проводник за секунду. Сопротивление уменьшает эту скорость. А напряжение - это разность потенциалов, и до меня не доходит что это значит.
3. Как происходит самоиндукция? Почему, если в цепи есть катушка и лампочка, то лампочка загорится с задержкой? Помогите представить что происходит в цепи. Вот как я это вижу: при замыкании ключа в цепи появляется ток, ток в катушке создает магнитное поле, которое препятствует току, т.е. ток в цепи становится слабее, но со временем возрастает, а именно лампочка загорается с задержкой т.к. нить накаливания просто нагревается медленно из-за слабого, но возрастающего со временем тока. Верно ли мое представление?
4. Почему энергия конденсатора W = C*U^2 / 2 зависит от напряжения которое его заряжало, опять же, может я не понимаю из-за непонимания сути напряжения, поэтому, еще раз прошу разжевать очень сильно. Как я вижу конденсатор: он просто хранит электроны, которые могут совершить работу, но причем тут напряжение, которое зарядило конденсатор?
>> No.158787 Reply
Господа физики, поясните гуманитарию, что вы имеет ввиду, когда говорите, что "до большого взрыва времени не было"? Штука ведь в том, что его уже нет. Время - это категория от человека, в реальности если и можно о чем-то говорить, то только об одном большом настоящем моменте, постоянном становлении и изменении материи. Прошлое - это память, будущее - это воображение. Ничего нет - нечему и становиться, следовательно, нет и времени.
Или есть какое-то отдельное, объективное время?
ускорение/замедление времени - это ни что иное, как ускорение/замедление становления материи?
>> No.158788 Reply
>>158782
1. По определению, от точки с большим потенциалом в точку с меньшим потенциалом бегут заряды. Заряды будут бежать, пока потенциалы точек не сравняются.
2. Представь, что ты берешь заряд и переносишь его из одной точки поля в другой. Ясно же, что при этом совершается какая-то работа. Так вот если поделить эту работу на заряд, то и получится величина, которая называется напряжением. Оказывается, напряжение для любых двух зарядов будет одинаковым. Если джоули считать рублями, а поездку из первой точки во вторую - авиаперевозкой, то напряжение - это цена перевозки одного килограмма из точки A в точку B, а работа - это общая стоимость перевозки.
3. В целом верно, подробнее написано у Мякишева в профильном уровне.
4. Конденсатор хранит энергию поля, заключенного между обкладками конденсатора. Если ты развернёшь формулу, то там вылезут только объём и напряженность, т.е. поле в чистом виде. От напряжения эта штука зависит потому, что напряжение - это энергетическая характеристика поля. Раз конденсатор хранит энергию поля, то эта энергия должна зависеть от энергетической характеристики поля, очевидно же. Гораздо любопытнее, почему там квадрат напряжения.
http://mathus.ru/phys/book.pdf

>>158787
Ты серьёзно? "Становление материи", втф.
>> No.158789 Reply
>>158788
> Становление материи"

Бытие - это становление, становление - это изменение.
>> No.158790 Reply
>>158789
Чё? Анон, говори нормально.
>> No.158792 Reply
>> No.158793 Reply
File: p0031.png
Png, 136.82 KB, 1202×1043 - Click the image to expand
edit Find source with google Find source with iqdb
p0031.png
>>158792
"Становление материи" не удовлетворяет критерию осмысленности. Пояснение.
1. Пикрелейтед.
2. https://en.wikipedia.org/wiki/Operational_definition
3. https://en.wikipedia.org/wiki/Mike_Alder#Newton.27s_flaming_laser_sword
Непонятно, про что идёт речь.

> время
http://rgho.st/7MYg7sbvS
Глава 1, параграфы с 1 по 3.
>> No.158794 Reply
File: MagrittePipe.jpg
Jpg, 34.78 KB, 670×519
edit Find source with google Find source with iqdb
MagrittePipe.jpg
File: часы.jpg
Jpg, 313.41 KB, 625×547
edit Find source with google Find source with iqdb
часы.jpg

>>158787
> когда говорите, что "до большого взрыва времени не было"
Кто тебе сказал, что физики так говорят?
> Время - это категория от человека,
Как и любое другое понятие. Карта -- не территория, а пикрелейтед -- даже не картина.
Есть модели, которые полезны до тех пор, пока дают верные предсказания.
> ускорение/замедление времени
У тебя есть вот такой столик со шкафом, который измеряет время. Ты помещаешь его куда-нибудь, где гравитация слабее (на самолёт подальше от земли), и обнаруживаешь, что часы стали идти быстрее.
Как ты это назовёшь, учитывая то, что работа шкафа зависит лишь от внутренных атомных процессов?

>>158782
> Почему если соединить проводником 2 заряженных тела их потенциал станет равным?
Потому, что возникнет ток, который распределит заряды равномерно.
Впрочем, нужно чтобы тела были металлическими.
> Как прочувствовать нутром напряжение?
https://en.wikipedia.org/wiki/Hydraulic_analogy
> Вот как я это вижу: при замыкании ключа в цепи появляется ток, ток в катушке создает магнитное поле, которое препятствует току, т.е. ток в цепи становится слабее, но со временем возрастает, а именно лампочка загорается с задержкой т.к. нить накаливания просто нагревается медленно из-за слабого, но возрастающего со временем тока. Верно ли мое представление?
Примерно так. Только всё происходит одновременно, так что ничто не слабеет, просто медленно возрастает.
> Как я вижу конденсатор: он просто хранит электроны
Как огурцы, да? Ничего, что конденсатор разрядится при любой возможности, только замкни? Лишние носители заряда не особо уживаются друг с другом.
>> No.158795 Reply
>>158794
> и обнаруживаешь, что часы стали идти быстрее
Когда часы возвращаются на землю, ты обнаруживаешь, что они ушли немного вперёд относительно тех часов, что были на земле всё время.
Ну или часы отправляют сигналы с самолёта на землю.
>> No.158825 Reply
File: 1.jpg
Jpg, 597.95 KB, 2304×1728
edit Find source with google Find source with iqdb
1.jpg
File: 2.jpg
Jpg, 489.89 KB, 2304×1728
edit Find source with google Find source with iqdb
2.jpg
File: 3.jpg
Jpg, 189.11 KB, 864×1152
edit Find source with google Find source with iqdb
3.jpg
File: 4.jpg
Jpg, 151.37 KB, 864×1152
edit Find source with google Find source with iqdb
4.jpg
File: 5.jpg
Jpg, 137.61 KB, 810×1080
edit Find source with google Find source with iqdb
5.jpg

Анониманимус, расскажи мне про дифференциальные и интегральные приложения в физике. В матане все понятно, неопределенный интеграл это бесконечное множество ф-ций, производные которых равны данной, а определенный по Риману это предел сумм произведений отрезков дельта икс на значения функций из любой точки этих отрезков, при стремлении характеристики разбиения к нулю. Ну и несобственные, Дарбу все там понятно.
И в физике самые простые приложения мне понятны. Скорость это по определению производная радиус вектора, момент силы это производная момента импульса и т.д. Но даже здесь уже проблемы. И вот первый вопрос:
1) Почему элементарная работа это dA? То есть дифференциал. И почему этот дифференциал неполный? И самое главное, почему работа на каком-то отрезке (от времени 1 до 2, от пути 1 до 2, от радиус вектора 1 до 2 и т.д) это интеграл от 1 до 2 по dA? Интеграл по Риману я понимаю, а тут ни функции никакой, ни разбиений, ничего. Например, задача с первой картинки. Я почему-то сразу так сделал и получил правильный ответ, но если меня спросят - почему так, я ответить не смогу. Ну допустим, dA это мой маленький дельта икс, помноженный на Fx, то есть на значение функции из отрезка, т.е. одно из слагаемых Римана. Но что dx, что Fx выражены через t и когда я уже интегрирую полученное выражение.. становится непонятно.
Но это только вопрос касательно простейших приложений, все таки работа так определена, и я просто не понимаю это определение. Но когда дело доходит до статистической физике в молекулярке.
2)Вот, например, даже барометрическая формула. Допустим dp и dh это очень маленькие приращения соответствующих функций, я понимаю замены и как берутся такие интегралы, но почему все получается так как получается? Почему мы просто берем и интегрируем? Я же не могу просто что-то любое, очень малое x объявить как dx, а потом интегрировать.
3)Приложение статистики. Опять, почему мы просто обозначаем вероятности и количества случаев с результатом x (кстати, тут ещё один вопрос в вопросе, написано, что dNx это кол-во результатов x? разве не от x до x+dx?) пишутся как dPx, и dNx, а потом интегрируем, считая, что это дифференциалы?
>> No.158826 Reply
>>158825
Физики используют математические значки чтобы записывать физические идеи. Значительная часть физиков на вопрос чем интеграл по Риману отличается от интеграла по Лебегу тебя нахуй пошлет.
> Я же не могу просто что-то любое, очень малое x объявить как dx, а потом интегрировать.
Почему нет? Все непрерывно, дифференцируемо и интегрируемо. Ты же можешь используя интеграл найти объем шара? Отличий никаких.
> разве не от x до x+dx?
Формально, да. Хочешь формализма? Вперед и с песней. Физики хотят короткой записи, которая понятна другим физикам.
Там, где ты бы написал lim_{Δx->0}(f(x,Δx)) любой физик напишет f(x,dx) просто потому что это короче, а что на этот счет там себе выдумают математики, дело математиков. В задачах того уровня, которые ты привел, во всяком случае. Когда они работать со всякими теориями струн, у них там что-то переключается, и они для своих dx в тех же формулах находят труёвое математическое обоснование из какой-нибудь дифференциальной геометрии.
>> No.158827 Reply
>>158825
Физики понимают дифференциал в первую очередь как бесконечно малое приращение, и вообще пользуются бесконечно малыми как реально существующми вещами.
В принципе, в этом нет ничего плохого, в дифференциальной геометрии делают точно так же.
> Допустим, dA это мой маленький дельта икс, помноженный на Fx, то есть на значение функции из отрезка, т.е. одно из слагаемых Римана. Но что dx, что Fx выражены через t и когда я уже интегрирую полученное выражение.. становится непонятно.
Инвариантность формы дифференциала.
> Я же не могу просто что-то любое, очень малое x объявить как dx, а потом интегрировать.
Можно! Правда, x -- не любое, и даже не число.
Опять же, физики очень вольно пользуются бесконечно малыми, но чтобы более-менее строго (не наивно) делать точно так же в математике, нужен некоторый дополнительный инструментарий.
А так, -dp = ρh dh -- простейшее дифференциальное уравнение, решаемое разделением переменных.
> Опять, почему мы просто обозначаем вероятности и количества случаев с результатом x пишутся как dPx, и dNx, а потом интегрируем, считая, что это дифференциалы?
У нас есть формула для чего-то дискретного, мы просто заменяем в ней суммы на интегралы, дельты на дифференциалы, и... получаем формулу для чего-то непрерывного!
Магия, правда?
> кстати, тут ещё один вопрос в вопросе, написано, что dNx это кол-во результатов x? разве не от x до x+dx?
Так короче.
>> No.159516 Reply
>>155821
Ну, ее и решают численно и не дуют. Это аналитически ее фиг решишь. Ни о какой случайности в принципе речи нет: есть система дифуров, есть начальные условия -- все корректно и хорошо, решение существует и единственно где надо. Другое дело, что нельзя просто взять и найти это решение, а "стохастичность", видимо, состоит в возможности хаотического поведения.
>> No.159530 Reply
>>159516
Численное решение нельзя бесконечно продолжать на временной прямой? Чем дальше мы от начальных условий, тем принципиально хуже будет ответ, даже при корректных нач условиях. Верно?
>> No.159534 Reply
>>159530
Во-первых, ошибка накапливается, это верно, во-вторых, я не совсем понимаю насчёт бесконечного продолжения: численно ты получаешь не формулу, а набор положений точек в зависимости от времени. Если просто численно считать -- то это займёт бесконечное время =)

В то же время, не все так плохо, можно подбирать помельче шаг интегрирования, выбирать численные методы, которые не сильно накапливают ошибку в энергии -- целая отдельная наука. Если считать какую-нибудь устойчивую траекторию, то можно вполне насчитать, что тела будут крутиться по ней до бесконечности
>> No.159540 Reply
File: 14937174510650.jpg
Jpg, 955.01 KB, 1920×1080 - Click the image to expand
edit Find source with google Find source with iqdb
14937174510650.jpg
Готовлюсь к поступлению в вуз, в списке вступительных экзаменов есть физика, которую я вообще не пытался учить ни в школе, ни в шараге. С чего вообще начать, чтобы нормально подготовиться к экзу?
>> No.159542 Reply
>>159540
Скачать методички ФИПИ за этот год и прорешать фипишные задачники, очевидно.
>> No.159901 Reply
>>159540
Некоторые картинки с поста выше - Савельев. Пока что самый охуенный учебник по физике, что я видел. Ещё есть элементарная физика Ландсберга или как-то так. Там все намного более упрощенное но и объяснено максимально понятно. Его читал иногда, пока был в лицее.
>> No.159981 Reply
File: 8.png
Png, 476.84 KB, 843×477 - Click the image to expand
edit Find source with google Find source with iqdb
8.png
Анон, помоги мне, пожалуйста, разобраться с мощностью в цепи. Я физикой никогда не увлекался и всегда очень поверхностно ее изучал, когда было необходимо. Сейчас выскочил у меня на четвертом курсе матфака экзамен по схемотехнике, сижу разбираюсь. Это предисловие к тому, почему мой вопрос может оказаться глупым с точки зрения человека, который разбирается в теме. В общем-то, это даже не вопрос, а цепь рассуждений и я не уверен, верны ли они.
Вот у нас есть мгновенная, активная, реактивная и до кучи еще полная мощность. Меня интересует, может ли каждая из них принимать отрицательную величину.
Мгновенная мощность - это dw/dt. Получается, если энергия будет возвращаться в источник, а не идти в нагрузку, то получится отрицательная мгновенная мощность. Возможен ли такой вариант вообще? Можно рассмотреть ее как u*i, где u,i - мгновенные значения напряжения и тока. Тогда мгновенная мощность будет отрицательной, если ток и напряжение направлены в противоположные стороны, так? Опять же таки, такое возможно?
Теперь активная мощность. Выражается как (UmIm/2)cos(phi), phi - разность фаз напряжения и тока. Амплитуды будут положительны, это понятно. Тогда все зависит от угла разности фаз. Может ли он лежать во 2 и 3 четвертях? Для цепи только с реактивной нагрузкой получается угол сдвига фаз pi/2. Для цепи только с активной - 0. Получается, что за эти границы он выходить не будет?
Аналогично с реактивной мощностью.
Полная мощность - это произведение действующих значений тока и напряжения, а, значит, быть отрицательным не может, так? Ведь действующие значения выражаются как отношение амплитуды к корню из двух, т.е. всегда положительны.
>> No.159983 Reply
File: 2ByFnoZh-do.jpg
Jpg, 107.60 KB, 961×1080 - Click the image to expand
edit Find source with google Find source with iqdb
2ByFnoZh-do.jpg
>>159981
Мощность это просто произведение напряжения на силу тока. Смотря какой ток и напряжение учитывать, разные мощности получаются.
Напряжение никуда не направлено, есть разность потенциалов между двумя точками. Ток течет от большего потенциала к меньшему. Если путей для тока несколько, то он делится в соответствии с правилами Киохгофа и все такое.
Не представляю как мощность может быть отрицательной, извини.
Фаза насколько я помню лежит в области от 0 до пи (либо от -пи/2 до пи/2, хз). Если фаза больше/меньше, ее можно математическими операциями свести обратно в этот промежуток (все возможные значения синуса/косинуса лежат в этих интервалах).
>> No.159984 Reply
>>159981
> Меня интересует, может ли каждая из них принимать отрицательную величину.
А какой физический смысл у отрицательной мощности?
>> No.159985 Reply
>>159984
Строго говоря, мощность это просто скорость какого-то процесса. Есть слово "скорость" для скорости перемещения, и слово "мощность" для всего остального. Вот и думая, какой смысл у отрицательной скорости какого-то процесса? В цепи я так понимаю, скорость перехода электрической энергии (диссипация) в тепло.
>> No.159986 Reply
>>159983
>>159984
Ох... Просто один из вопросов к экзамену так и стоит:
> Какая из них может быть отрицательной и почему?
Потому я и пытаюсь как-то подогнать, какая из них может быть отрицательной.

> Напряжение никуда не направлено... Ток течет от большего потенциала к меньшему
Но значение тока может же записываться отрицательно, так (направление тока же произвольно выбираем при расчетах)? Если рассматривать участок некоторой цепи. И тогда на этом участке мощность может быть отрицательной? Ну, с точки зрения математики по крайней мере. Может быть, что-то такое подразумевается в вопросе.

> А какой физический смысл у отрицательной мощности?
Ммм... Энергия, передаваемая в приемник со временем уменьшается. Получается, что она должна возвращаться из приемника в источник. Чушь какая-то получается, да.
>> No.159987 Reply
File: 2017-05-27_21-46-...
Png, 23.26 KB, 701×342
edit Find source with google Find source with iqdb
2017-05-27_21-46-46.png
File: ONOwWg-IB30.jpg
Jpg, 70.69 KB, 640×640
edit Find source with google Find source with iqdb
ONOwWg-IB30.jpg

>>159986
А все, я понял, анон. Мгновенная мощность может быть отрицательной. Ток может на пи/2 запаздывать от напряжения. Тогда ток будет записываться как косинус, а напряжение как синус. Произведение их будет пикрилейтед. Генерал ясен хуй, что мгновенная мощность тоже колеблется и пол периода является отрицательной.
>> No.159989 Reply
>>159986
> Чушь какая-то получается, да.
А чем источник отличается от приёмника?
>> No.159990 Reply
File: anime_dobra_tebe_390032.jpg
Jpg, 22.85 KB, 522×393 - Click the image to expand
edit Find source with google Find source with iqdb
anime_dobra_tebe_390032.jpg
>>159987
Спасибо!

Думаю, я к вам на кафедру вернусь еще в ближайшую неделю с другими вопросами, надеюсь, вы тут не против новичков в физике.
>> No.159991 Reply
File: oYMKiocHf08.jpg
Jpg, 35.80 KB, 604×360 - Click the image to expand
edit Find source with google Find source with iqdb
oYMKiocHf08.jpg
>>159990
Конечно, брат, заходи.
>> No.159993 Reply
>>159989
Я все думал, как тебе ответить и окончательно запутался в этом вопросе. Вот мой краткий ответ:
Источник генерирует ток (напряжение), а приемник потребляет/запасает/преобразует в другие виды энергии.
А потом я придумал пример с батарейкой, которая приемник когда заряжается и источник, когда вставлена в часы. После чего у меня появились вопросы о том, можно ли считать конденсатор, который высвобождает запасенную энергию источником и прочее, в итоге я запутался.
>> No.159994 Reply
Хотел погуглить баланс мощностей, но не нашёл ничего норм.
Почему все сайты про физику в интернете настолько убоги? Что нибудь годное про алгебраическую топологию или теоркат можно найти за секунды, не говоря уже про информатику и программирование, когда же выдача гугла даже на тривиальные физические вопросы даёт тонны мусора.
>> No.160008 Reply
File: Безымянный.png
Png, 8.10 KB, 208×196 - Click the image to expand
edit Find source with google Find source with iqdb
Безымянный.png
Нефизик снова в треде!
На этот раз я снова с очень нубским вопросом: как читать схемы? Объясню свои проблемы с этим на примере простейшего ФНЧ с пикчи.
В моем воображении электроны появляются одновременно на двух входных зажимах (1,2) и оттуда текут к двум выходным зажимам (3,4). Вот по этим нарисованным проводкам. Но тогда они будут проходить на оба выхода при любых частотах!
Пусть подается ток на низкой частоте. На верхней части схемы ток входит в фильтр с зажима 1, удачно проходит через индуктивность и выходит на зажим 3. А между 2 и 4 зажимами вообще ничего нет.
На высокой частоте просто ток будет со 2 зажима разделяться и часть его удачно пройдет через емкость к зажиму 3, а часть к 4.
Я догадываюсь, что наверняка допускаю какую-то очень глупую ошибку в попытках понять поток на схеме, но не знаю, где именно эта ошибка. Пытался нагуглить каких-нибудь анимаций с этим процессом, но не получилось.
В общем, объясните мне, пожалуйста, господа, что значат эти входы/выходы на схемах и как тут бегают электроны.
>> No.160009 Reply
>>160008
> На высокой частоте просто ток будет со 2 зажима разделяться и часть его удачно пройдет через емкость к зажиму 3, а часть к 4.
Нет. 2 и 4 - это принципиально одна точка. Ток из нее никуда не пойдет в принципе. Если у тока есть прямой путь, то он даже не попытается влезть на сопротивление, активное или реактивное.
В зависимости от частоты здесь меняется баланс напряжения между 1-3 и 3-4. Сопротивление катушки нарастает, сопротивление конденсатора понижается. На постоянном токе через конденсатор не идет ничего, напряжение 3-4 равно напряжению 1-2. На частоте, уходящей в бесконечность, конденсатора считай что нет, точки 3 и 4 стягиваются в одну, напряжения на выходе нет.
Представь переходный процесс - сначала у нас нет напряжения на 1-2, и мы начинаем его подавать. Ток начинает течь через катушку для того чтобы зарядить конденсатор. В ней из-за этого постепенно нарастает магнитное поле, придавая ей самой реактивное сопротивление. На катушке, запаздывая за U1-2 будет появляться напряжение U1-3, тем самым не давая конденсатору получить все U1-2 на себя. Однако, когда U1-2 пройдет пик и начнет спадать, магнитное поле катушки ослабевая начнет само производить ток (ЭДС самоиндукции), как бы обеспечивая инерцию, и этот ток будет дозаряжать конденсатор дальше, до максимального уровня который был на U1-2, но теперь уже спал.
Фишка полосы пропускания здесь в том, что конденсатор при этом будет еще и разряжаться в нагрузку на 3-4. Если он будет слишком маленькой емкости (для данной частоты), то и разряжаться он будет очень быстро, а значит "медленный" ток низкой частоты просто зарядит конденсатор до уровня 1-2 и попрет в нагрузку. Однако, если повышать частоту, то она входное напряжение будет меняться быстрее, чем может зарядиться конденсатор, а быстро меняющееся поле катушки будет давать высокое реактивное сопротивление на ней. То есть, на больших частотах ток течь не будет, а напряжение на конденсаторе будет околонулевым (или таким, на которое его предварительно зарядили, но на таких схемах нас это как правило не интересует).
>> No.160010 Reply
File: U.png
Png, 1179.05 KB, 993×751 - Click the image to expand
edit Find source with google Find source with iqdb
U.png
>>160009
Спасибо!
>> No.160029 Reply
File: _20170603_143858.JPG
Jpg, 1010.76 KB, 3920×2204
edit Find source with google Find source with iqdb
_20170603_143858.JPG
File: DSC_0610.JPG
Jpg, 1750.10 KB, 3920×2204
edit Find source with google Find source with iqdb
DSC_0610.JPG

В руки попал поляроид, решил поизучать поляризацию неба. Вчера получилось таки поймать. Облака не поляризованы, только небо. (это от формы молекул воды или ещё от чего-то?) Плоскость, откуда идет поляризованый свет, перпендикулярна направлению на солнце. Сильнее в зените, слабее на горизонте (тоже еще не понял, почему).
>> No.160031 Reply
>> No.160043 Reply
Я продолжаю погружаться в физику, а точнее, в электронику.
Вот есть выпрямительный диод. Можно его присоединить к схеме прямым включением(+ к аноду, - к катоду), а можно обратным. И при обратном подключении ток просто не пойдет. Так вот, в обратном подключении выпрямительного диода есть смысл какой-то для реальных схем?
>> No.160046 Reply
File: Misaka_Mikoto_by_Ray_Dump.png
Png, 208.89 KB, 1219×656 - Click the image to expand
edit Find source with google Find source with iqdb
Misaka_Mikoto_by_Ray_Dump.png
>>160043
Не пойдет ток постоянный при обратном. Переменный переменный потеряет половину периода (какую именно половину зависит от включения диода). Если диодов несколько (можно и с одним, но неэффективно), то можно сделать так, что переменный ток станет постоянным (гугли выпрямители тока, диодный мост).
Еще можно такое придумать применение. Например, тебе нужно зарядить конденсатор (для гаусс-пушки ага), ты включаешь его прямо, чтобы ток с батареи шел на конденсатор, а с конденсатора обратно не утекал (для конденсатора это будет обратное включение). На самом деле тип включения зависит только от направления тока. Не знаю, ответил ли на твой вопрос, было довольно туманно.
>> No.160047 Reply
File: slide-9.jpg
Jpg, 49.38 KB, 1024×574 - Click the image to expand
edit Find source with google Find source with iqdb
slide-9.jpg
>>160046
Да, ответил, спасибо!
И можно тогда вдогонку вопрос по диодам? Вот есть туннельный диод. И вот я прочитал о нем в одной статье, в другой, в третьей, но не уверен, что понял его применение. Я так понимаю, что его рабочее напряжение - это участок U1-U2. И при работе на этом участке он будет усиливать проходящие через него сигналы за счет отрицательного сопротивления. Это верно или я таки не прав?
>> No.160052 Reply
File: Tunnel_diode_ampl...
Png, 34.38 KB, 1280×982
edit Find source with google Find source with iqdb
Tunnel_diode_amplifier.png
File: Tunnel_diode_ampl...
Png, 67.66 KB, 1103×967
edit Find source with google Find source with iqdb
Tunnel_diode_amplifier_graph.png

>>160047
> усиливать проходящие через него сигналы за счет отрицательного дифференциального сопротивления
>> No.160054 Reply
>>160052
Угу, благодарю.
>> No.160109 Reply
File: 1938full.jpg
Jpg, 64.70 KB, 800×600 - Click the image to expand
edit Find source with google Find source with iqdb
1938full.jpg
Бамп >>141498-задаче!
Как такое вообще решать? Как от силы, действующей на заряд в поле, перейти к энергии выхода электрона?
>> No.160114 Reply
>>160109
Работа равна произведению напряжения на заряд.
>> No.160122 Reply
>>160114
О каком напряжении идет речь? У нас тут не потенциальное электрическое поле. У нас тут сила Лоренца, действующая на электроны и на ядро.
>> No.160203 Reply
File: 14742240170620.jpg
Jpg, 16.10 KB, 344×326 - Click the image to expand
edit Find source with google Find source with iqdb
14742240170620.jpg
Я спокойно изучал себе химию и всё шло гладко, но тут я наткнулся на одну темку:"Квантовая модель строения атома". Ну, подумал я, сейчас всё будет как и с другими темами, но, наткнувшись на корпускулярно-волновую природу атома", я понял, что ничего не понял. Дальше хуже - соотношение де Бройля, принцип Паули, спины атомов и т.д.
ШТО ЕНТО ЗНАЧИТ Я НЕ ПОНИМАТ!
Дайте какие-нибудь наглядные пособия(а лучше, конечно, видео) по этой теме, чтобы я хотя бы это понял. Школьные знания по физике у меня просто на ноле, так что как можно проще, пожалуйста
>> No.160206 Reply
>>160203
Наглядной модели для этого нет. Есть хитро закрученный матан, устроенный как черный ящик. На вход этому матану подаются текущие показания измерительного прибора, матан их обрабатывает и говорит, какими будут показания измерительного прибора в будущем. Что внутри черного ящика - науке неизвестно.
>> No.160208 Reply
File: -834.jpg
Jpg, 21.74 KB, 480×360 - Click the image to expand
edit Find source with google Find source with iqdb
-834.jpg
>>160203
Очень трудно помочь в твоем случае. Читай что есть, нормальные учебники то есть, ютуб идет в пизду. Если есть будут конкретные вопросы, то обращайся сюда. Я уверен, что ты сможешь превозмочь. Это конечно не худлит читать, но ты умный анон, ты сможешь. А если что тут поможем.
>> No.160217 Reply
>>160208
Спасибо, анон, хоть чем то помог. На днях углублюсь в физику, а потом уже задам вопросы.
>> No.160218 Reply
File: 2017-05-15_20-14-24.jpg
Jpg, 12.19 KB, 398×338 - Click the image to expand
edit Find source with google Find source with iqdb
2017-05-15_20-14-24.jpg
Хочу опять мудрую на ОП-пик.
>> No.161183 Reply
>>160109
Может, без этой энергии решать? Я атомы еще не изучал, так что дальше может идти ересь, вы предупреждены.
Просто решить как для модели гирьки. Момент отрыва обозначить через приравнение силы сцепления атомов к центробежному ускорению.
>> No.161648 Reply
>>160109
Энергия системы зарядов в однородном магнитном поле складывается из произведения магнитных моментов на напряженность поля и выражения, квадратичного по полю. Тебе необходимо, чтоб энергия за счет взаимодействия с полем оказалась больше пределов ионизации гелия.
>> No.165223 Reply
Мне нужно рассчитать трос на разрыв,то есть максимальное ускорение которое он может выдержать.Я уже пытался смотреть в учебнике сопромата но там только балки и перекрытия.Где можно почитать про физику гибких тел?И как вообще их моделировать?Я думаю моделировать веревку как систему из связанных колец.Насколько справедлива такая модель?
>> No.165343 Reply
Доброго времени.
Не поможете, пожалуйста, с задачкой?
Дано: Шарик (80г.) на нити (0.2м). Движется по окружности с постоянной частотой, опираясь на горизонтальную плоскость. При увеличении длины нити вдвое частота равна 6-ти об/c. Трением о плоскость пренебрегаем. Надо найти работу при удлинении нити и первоначальную частоту.
Хотя бы на наводку, если можно.


Password:

[ /tv/ /rf/ /vg/ /a/ /b/ /u/ /bo/ /fur/ /to/ /dt/ /cp/ /oe/ /bg/ /ve/ /r/ /mad/ /d/ /mu/ /cr/ /di/ /sw/ /hr/ /wh/ /lor/ /s/ /hau/ /slow/ /gf/ /vn/ /w/ /ma/ /azu/ /wn/ ] [ Main | Settings | Bookmarks | Music Player ]